SAER 2003 Flashcards

1
Q

A 61-year-old woman reports tingling in the posterior aspect of her upper arm, forearm and hand. She has no history of trauma and radiographs are negative. Her physician has ruled out a cervical
radiculopathy by electrodiagnostic studies, magnetic resonance imaging, and physical examination. The physician is suspicious that the cause of the patient’s paresthesias is myofascial in nature. Which finding would be consistent with myofascial pain?
(a) Positive sharp waves on electromyography at rest in trigger points
(b) Reproduction of symptoms with palpation over the trigger point
(c) Elevated serum creatine phosphokinase associated with prolonged muscle activity
(d) Resolution of symptoms with isokinetic exercise of the affected muscle

A

(b) Moderate, sustained pressure on an irritable trigger point causes symptoms in the reference zone for
that muscle. Myofascial trigger points are electrically silent and show no resting muscle activity on
EMG. No elevation in CPK is seen with this condition. Local injection and/or spray and stretch of
the muscle are the treatments of choice. Isokinetic exercise is not indicated as a treatment for this
condition.

How well did you know this?
1
Not at all
2
3
4
5
Perfectly
2
Q

A 24-year-old man was in a motor vehicle collision 36 hours ago. His initial Glasgow Coma Scale
score was 13 and his initial head computed tomography scan showed a small frontal contusion.
Initial blood alcohol level was .15g/dL. He is currently disoriented, combative, and tachycardic. He
reports visual hallucinations. Management of this case should include
(a) benzodiazepines for alcohol withdrawal.
(b) anticonvulsants for agitation.
(c) neuroleptics to treat hallucinations.
(d) beta-blockers to treat tachycardia.

A

(a) Premorbid alcohol abuse is commonly seen in people sustaining brain injury and alcohol
withdrawal causes agitation and hallucinations.

How well did you know this?
1
Not at all
2
3
4
5
Perfectly
3
Q

What bathroom modification should be made for nonambulatory individuals who have the ability to
transfer themselves in and out of a wheelchair?
(a) Toilet height of 15 inches
(b) Cabinet heights of 36 inches from the floor
(c) Three feet of clear turning space in the bathroom
(d) Separate hot and cold water handle controls

A

(b) When modifying a bathroom for a person who has the ability to transfer him/herself in and out of a
wheelchair, it is important to design bathroom space for the person’s efficiency, to allow him or her
to be as independent as possible. This includes providing a minimum of 5 feet of clear turning
space and toilets at least 20 inches high (it is reasonable to use a standard height toilet with a raised
toilet seat). Cabinets should be positioned for easy access. In most cases, the bottom of the cabinet
should be 36 inches from the floor. Mirrors also should be positioned so that their bottom edge is 36
inches from the floor. Single-lever type handles should be used for water control. The use of a
single handle control with a lever or blade shape that mixes the water to control and adjusts the
flow is recommended. Temperatures should be set between 110 and a 120 degrees Fahrenheit at the
point of supply so that the delivery temperature will be approximately 105 degrees Fahrenheit. All
hot water feed and drainage pipes should be insulated to prevent scalding.

How well did you know this?
1
Not at all
2
3
4
5
Perfectly
4
Q

A head and neck cancer patient completed external beam radiation therapy 1 week ago. You inform
the patient that prevention of cervical soft tissue contractures requires daily cervical range-of-motion
exercises for at least
(a) 1 month.
(b) 6 months.
(c) 1 year.
(d) 5 years.

A

(d) The late effects of external beam radiation therapy can continue for at least 5 years. Fibrosis of
fascia, skin, and muscles following radiation therapy is mediated by radiation-induced
microvascular injury. Daily cervical range-of -motion exercises should be performed for at least 5
years after completion of treatment to insure that progressive fibrosis does not occur.

How well did you know this?
1
Not at all
2
3
4
5
Perfectly
5
Q

Which finding on your history and physical examination of an injured worker with low back pain
would require immediate intervention?
(a) Pain radiating into the posterior thigh
(b) Pain that awakens the worker at night
(c) Paresthesias in the great toe
(d) Ankle weakness only with long-distance walking

A

(b) Pain that awakens the worker at night may indicate a serious etiology including tumor or infection.
Radicular pain, paresthesias, and weakness with fatigue may also present with serious as well as
common causes such as disc herniations and spinal stenosis. The latter may help direct the
diagnosis. However, pain worse at night should alert the treating physician to evaluate more serious
etiology without delay.

How well did you know this?
1
Not at all
2
3
4
5
Perfectly
6
Q

Which radial innervated muscle is innervated by the C5 root?

(a) Anconeus
(b) Extensor carpi radialis longus
(c) Brachioradialis
(d) Triceps

A

(c) None of the other muscles listed receives C5 innervation. The supinator is the only other radial
innervated muscle that has C5 innervation.

How well did you know this?
1
Not at all
2
3
4
5
Perfectly
7
Q

In children with spastic cerebral palsy, which intervention strengthens weak muscles?

(a) Ankle-foot orthotics
(b) Tendon transfer surgery
(c) Intrathecal baclofen
(d) Functional training program

A

(d) Children with cerebral palsy often have weakness as part of their disorder. Treatments such as
bracing, tendon lengthening or transfers, and medications such as botulinum toxin or intrathecal
baclofen add to this weakness. Strengthening programs or functional training programs can help to
strengthen weak muscles.

How well did you know this?
1
Not at all
2
3
4
5
Perfectly
8
Q

Shoulder pain in the hemiparetic shoulder due to cerebrovascular accident

(a) is commonly due to formation of heterotopic ossification in the glenohumeral joint.
(b) occurs more frequently in flaccid hemiplegia than in spastic hemiplegia.
(c) results in complex regional pain syndrome in up to 10%–15% of patients.
(d) is best treated with rest and immobilization in a sling while in bed.

A

(c) Complex regional pain syndrome is present in up to 10%–15% of patients with stroke and shoulder
pain. This condition has also been called shoulder-hand syndrome and reflex sympathetic
dystrophy. Heterotopic ossification may occur in the elbow or shoulder joint following CVA, but it
is uncommon. Shoulder pain associated with hemiparesis or hemiplegia due to stroke is more often
associated with spastic hemiparesis than with flaccid hemiparesis. Gentle, passive range of motion
exercises should be started early. Proper positioning should begin early as well. Abduction and
external rotation is the position of choice while the patient is in bed. A sling should not be used in
bed.

How well did you know this?
1
Not at all
2
3
4
5
Perfectly
9
Q

What is the most common form of arthritis in adults?

(a) Crystal arthritis
(b) Septic arthritis
(c) Osteoarthritis
(d) Rheumatoid arthritis

A

(c) Osteoarthritis is the most common form of arthritis.

How well did you know this?
1
Not at all
2
3
4
5
Perfectly
10
Q

A 59-year-old man admitted with atypical chest pain underwent an angiogram 2 days ago. He is now
having difficulty walking. Physical examination demonstrates 2/5 strength in right hip flexion and
knee extension. Muscle strength is otherwise 5/5. Sensation is decreased over the medial distal right
leg. What initial diagnostic test would be most appropriate?
(a) Electromyogram and nerve conduction velocities
(b) Magnetic resonance imaging of the lumbosacral spine
(c) Computed tomography scan of the pelvis
(d) Ultrasound of the abdomen

A

(c) The most likely pathology is the involvement of the femoral nerve secondary to hemorrhage. In
this situation imaging studies should be done promptly to assess for a hematoma or a mass.
Electrodiagnostic studies are useful to confirm the presence of femoral nerve involvement, but one
should wait 3–4 weeks postinjury.

How well did you know this?
1
Not at all
2
3
4
5
Perfectly
11
Q

What function is expected in an individual with a C7 ASIA class A spinal cord injury?

(a) Need assistance to perform level transfers
(b) Pressure reliefs primarily by side-to-side weight shift
(c) Independence in bowel and bladder management
(d) Independent dressing and bathing with adaptive equipment

A

(d) For persons with motor level C5, activities of daily living include drinking from a cup and feeding
with static spoons and set-up, some oral/facial hygiene, writing and typing with equipment, and
possibly some upper-body dressing. At the C6 injury level, individuals are able to feed and perform
upper body dressing with set-up and can perform level surface transfers with assistance. Persons
with motor level C7 ASIA class A should be able to independently feed, dress, and bathe
themselves, using adaptive equipment and built up utensils. They should be independent with bed
mobility, and level surface transfers and should be able to propel a wheelchair outdoors.
Independence in bowel and bladder function is generally seen with injury at level T1 and lower.

How well did you know this?
1
Not at all
2
3
4
5
Perfectly
12
Q

During discussion with the parents of a 2-year-old leukemic patient, you inform them that significant
brain irradiation almost uniformly produces
(a) attention deficits.
(b) focal motor weakness.
(c) ataxia.
(d) anosmia.

A

(a) An adverse sequela of intensive pediatric anticancer therapy is learning difficulty. Impaired
learning can exert a deleterious long-term impact. Whole brain irradiation for leukemic prophylaxis
results in enlarged cerebral sulci and ventriculomegaly on cranial imaging.Clinical
symptomatology roughly correlates with scan findings. Virtually all patients in whom a substantial
portion of the brain is radiated complain of memory loss and attentional deficits.

How well did you know this?
1
Not at all
2
3
4
5
Perfectly
13
Q

Why is it important to ask the injured worker if he/she is involved in litigation against the
workplace?
(a) It allows the physician to deny seeing the patient to avoid a deposition.
(b) It is not an appropriate question for the physician to ask.
(c) Workers with attorneys involved are less likely to return to work.
(d) The employer will be more cooperative in facilitating return to work.

A

(c) Several studies have found a strong relationship between attorney involvement and not returning to
work. In workman’s compensation cases involving lawyers, 73% of injured workers did not return
to work. In cases without lawyer involvement, 32% did not return to work. These factors are
important for the physician to know initially to best understand the psychosocial barriers involved
in treating the injured worker.

How well did you know this?
1
Not at all
2
3
4
5
Perfectly
14
Q

Which myopathy is most likely to demonstrate a “normal” EMG?

(a) Myotonic dystrophy
(b) Polymyositis
(c) Steroid myopathy
(d) Duchenne muscular dystrophy

A

(c) In steroid myopathy the only abnormalities are atrophy of the type II fibers. Since these fibers are
recruited last, when the screen is full of motor units, it is usually difficult to appreciate subtle
amplitude changes. The other myopathies noted typically may exhibit positive waves and
fibrillations with motor unit changes in the type I fibers.

How well did you know this?
1
Not at all
2
3
4
5
Perfectly
15
Q

One of your 4-year-old patients exhibits the following characteristics: distress over minor changes in
environment, echolalia, lack of awareness of the existence of feelings in others, nonparticipation in
simple games. The most likely diagnosis is
(a) autism.
(b) cerebral palsy.
(c) hearing impairment.
(d) mental retardation.

A

(a) Autism is characterized by echolalia, inability to play reciprocally, and abnormal relationships with
people. While children with mental retardation, cerebral palsy, and hearing impairment may have
some of these features, they do not have all of them in the absence of autism.

How well did you know this?
1
Not at all
2
3
4
5
Perfectly
16
Q

After aspirating 10cc of cloudy fluid from a patients knee, you find a white blood cell count of
20,000/cc, with intracellular, negatively birefringent rod-shaped structures under the polarized light
microscope. Your diagnosis is
(a) septic arthritis.
(b) gout.
(c) pseudo-gout.
(d) anterior cruciate ligament tear.

A

(b) A septic joint would reveal a white blood cell count greater than 50,000/cc, pseudogout has
positive-birefringent crystals, an anterior cruciate ligament tear would reveal a bloody aspirate.
Urate crystals from gout are negatively birefringent, needle- or rod-shaped crystals that can be
intracellular.

How well did you know this?
1
Not at all
2
3
4
5
Perfectly
17
Q

A 17-year-old boy from India presents with a longstanding history of areflexia and asymmetric
muscular atrophy after a febrile illness as a child. The likeliest site of neurologic pathology is
(a) myelin sheath of peripheral nerves.
(b) axons of peripheral nerves.
(c) anterior horn cells.
(d) muscle membrane.

A

(c) Poliomyelitis involves the anterior horn cells. During an acute infection, the virus is transported to
the anterior horn cells followed by inflammation and loss of spinal and bulbar motor neurons.

How well did you know this?
1
Not at all
2
3
4
5
Perfectly
18
Q

Regarding spinal shock in acute spinal cord injury,

(a) duration of spinal shock is correlated with long term outcome.
(b) reflex activity typically returns over the course of days.
(c) a reliable ASIA classification can be performed during spinal shock.
(d) it is more common in tetraplegia than in paraplegia.

A

(c) Spinal shock is a condition in which upper motor neuron sensory motor loss is associated with
areflexia below the level of injury. It is a poorly defined phenomenon. Reflex activity can often be
detected by electrophysiologic study when it is not clinically apparent. Reflex activity typically
returns over the course of weeks or months. The presence of spinal shock is of marginal prognostic
significance. A reliable ASIA classification can be carried out when spinal shock is present.

How well did you know this?
1
Not at all
2
3
4
5
Perfectly
19
Q

Which situation is associated with good treatment outcome in the injured worker?

(a) Poor evaluation by the employer within the past 6 months
(b) The employee works in middle management
(c) The employee received a recent increase in salary
(d) Communication between the worker and supervisor is empathetic

A

(d) Good treatment outcome is associated with a history of personal empathetic communication
between the supervisor and injured employee. No data show improved outcomes in injured workers
who have recently received a raise or have worked for the employer more than 5 years. Poor
outcomes have been found in injured workers who have received a poor evaluation within the 6
months preceding the injury. The level of position within the workplace is irrelevant in a worker’s
compensation outcome.

How well did you know this?
1
Not at all
2
3
4
5
Perfectly
20
Q

Which electrodiagnostic finding is more common in radiation plexopathy than in neoplastic
plexopathy?
(a) Myokymic discharges
(b) Fibrillations
(c) Decreased motor unit recruitment
(d) Decreased amplitude of the sensory nerve action potential

A

(a) Myokymia is present in 50% of patients who have radiation plexopathy, but is rarely seen in
neoplastic plexopathy. The other findings noted in both plexopathies, but predominate in neither.

How well did you know this?
1
Not at all
2
3
4
5
Perfectly
21
Q

Which finding is normal in newborn infants?

(a) Extensor tone predominates
(b) Hands are kept fisted
(c) Spine is straight when held in sitting position
(d) Unable to turn head to side in prone position

A

(b) In normal newborn infants flexor tone predominates and hands are kept fisted. In prone position a
normal newborn is able to turn the head to either side. The newborn has a rounded spine when
placed in supported sitting.

How well did you know this?
1
Not at all
2
3
4
5
Perfectly
22
Q

A 16-year-old male with a right above-knee amputation presents to your clinic to get your opinion on
operating a motor vehicle. You advise him that he would be required to
(a) strengthen his hip extensors.
(b) change the position of the car’s accelerator and brake.
(c) obtain a special driving prosthesis.
(d) install a handbrake on the vehicle’s left floor.

A

(b) The only situation that applies in this scenario is changing the position of the pedals in order to
operate the vehicle with the left lower limb. Installing a handbrake is reserved for persons with right
upper limb amputations. Other acceptable recommendations could include automatic transmission
and/or hand controls.

How well did you know this?
1
Not at all
2
3
4
5
Perfectly
23
Q

Which statement regarding cervical traction is correct?
(a) The best angle of pull is between 10° and 20° of cervical extension.
(b) Its use in patients with rheumatoid arthritis is absolutely contraindicated.
(c) At least 25 pounds of force is necessary to counter the effects of gravity on the head.
(d) With an over-the-door traction unit, the patient should face away from the door to which the
pulley is attached.

A

(b) Patients with rheumatoid arthritis have ligamentous instability. This can lead to subluxation of
cervical vertebrae, especially at the atlantoaxial joint (C1-2). Because instability can lead to spinal
cord compression, cervical traction is, therefore, absolutely contraindicated in persons with
rheumatoid arthritis. The best angle of pull is between 20° and 30° of flexion. The most common
reason for cervical traction to fail or to exacerbate symptoms is applying the force in extension
rather than in flexion. The home traction unit should always be placed so the patient is facing
toward the door to which the pulley is attached. At least 10 pounds of force is needed to counter the
effects of gravity on the head. To straighten the cervical lordotic curve requires 25 pounds of force.

How well did you know this?
1
Not at all
2
3
4
5
Perfectly
24
Q

According to the Hunt and Hess Scale, which grade of subarachnoid hemorrhage would apply to a
patient who presents with moderately severe headache, meningismus, and cranial nerve deficit?
(a) 0
(b) 1
(c) 2
(d) 3

A

(c) Grade 2 of the Hunt and Hess Scale is moderately severe headache/meningismus, no neurologic
deficit, except cranial nerve palsy.

How well did you know this?
1
Not at all
2
3
4
5
Perfectly
25
Q

Mr. Jones comes to your office complaining of a hot, painful, swollen left wrist. Additional findings
on physical examination include swan-neck and boutinierre deformities, subluxation of the
metacarpophalangeal joints with ulnar deviation of the digits. What will x-ray findings of his wrist
reveal?
(a) Chondrocalcinosis of articular cartilage
(b) Severe marginal erosions with juxta-articular osteopenia
(c) Bony erosion with an overhanging edge
(d) Pencil-in-cup deformity

A

(b) This patient has rheumatoid arthritis which shows juxta-articular osteopenia. Gout characteristically
reveals “overhanging edge” lytic lesions. Chondrocalcinosis is seen in pseudo-gout, and pencil-incup
deformity is seen with psoriatic arthritis.

How well did you know this?
1
Not at all
2
3
4
5
Perfectly
26
Q

What is the most common initial manifestation in Parkinson’s disease?

(a) Fatigue
(b) Gait difficulty
(c) Hypophonia
(d) Resting tremor

A

(d) Resting tremor and bradykinesia are the most common initial manifestations of Parkinson’s disease.
The other signs are less likely to be presenting complaints in parkinsonism.

How well did you know this?
1
Not at all
2
3
4
5
Perfectly
27
Q

Which chemotherapeutic agent can produce sensory polyneuropathy?

(a) Paclitaxel (Taxol)
(b) Doxorubicin (Adriamycin)
(c) Cyclophosphamide (Cytoxan)
(d) 5-fluorouracil (5-FU)

A

(a) The taxanes, docetaxel (Taxotere) and paclitaxel (Taxol), are being used increasingly as first line
chemotherapy for a variety of malignancies including ovarian, breast, and head and neck cancers.
They are associated with a high incidence of sensory polyneuropathy. Fortunately, this often
resolves or is significantly diminished following discontinuation of the agent. However, some
patients continue to experience severe, debilitating neuropathy.

How well did you know this?
1
Not at all
2
3
4
5
Perfectly
28
Q

What work-place situation is the most frequent cause of low back pain in workers?

(a) Jobs that result in falls at work
(b) Jobs that require standing for more than 4 hours
(c) Jobs that requires lifting and material handling
(d) Jobs that require sitting more than 2 hours

A

(c) Jobs that require lifting and material handling place the worker at increased risk for low back
injury. Lifting frequency, load movement, trunk twisting and trunk sagittal angle predict medium
and high-risk occupational low back pain. No risk correlation has been found for length of time
sitting or standing. Although falls in the work place put the worker at risk for low back injury, the
repetitive motion involved in lifting and material handling is thought to cause low back pain more
frequently.

How well did you know this?
1
Not at all
2
3
4
5
Perfectly
29
Q
Where do you place the cathode when performing a median motor nerve conduction study,
stimulating at the elbow?
(a) Lateral to the biceps tendon
(b) Medial to the brachial artery
(c) Lateral to brachioradialis
(d) Medial to brachioradialis
A

(b) The median nerve is just medial to the brachial artery at the elbow.

How well did you know this?
1
Not at all
2
3
4
5
Perfectly
30
Q

The family of your 10-year-old patient who had a severe traumatic brain injury 6 weeks ago asks you
if they may feed their son. You observe that he is agitated at times, has a hoarse voice, and drools.
You try to feed him applesauce and notice that he seems to swallow part of it and does not cough.
The most likely finding on the videofluoroscopic feeding study will be
(a) Silent aspiration.
(b) Reflux.
(c) Coughing and gagging.
(d) Normal swallow.

A

(a) The lack of coughing in a patient with neurologic impairment when he/she is presented with food
may mean a normal swallow, but is more likely to mean silent aspiration. A normal
videofluoroscopic swallowing study is unlikely in a patient with a traumatic brain injury (TBI) who
is drooling and hoarse. Hoarseness may be a sign of reflux, but in a child with a TBI it is more
likely to mean vocal cord abnormality.

How well did you know this?
1
Not at all
2
3
4
5
Perfectly
31
Q

The criterion scale used to describe severity of brain injury is the

(a) Disability Rating Scale.
(b) Agitated Behavior Scale.
(c) FIM™ instrument.
(d) Glasgow Coma Scale.

A

(d) The criterion to describe the severity of a traumatic brain injury is the Glascow Coma Scale (GCS).
GCS score of 13-15 = mild
GCS score of 9-12 = moderate
GCS score of 3-8 = severe

How well did you know this?
1
Not at all
2
3
4
5
Perfectly
32
Q

A 40-year-old woman with a history of irritable bowel syndrome and tension headaches complains of
increasing fatigue and diffuse muscle soreness in her neck, shoulders, and low back. She has a
nonfocal neurologic examination. Initial recommendations should include
(a) craniosacral manipulation.
(b) closed kinetic chain exercises to strengthen the shoulder girdle.
(c) electrical stimulation to the upper and mid back.
(d) walking on a daily basis

A

(d) Promoting a restorative sleep and 20–30 minutes of aerobic activity daily is recommended to treat
fibromyalgia. Passive modalities are not the optimal therapeutic intervention.

How well did you know this?
1
Not at all
2
3
4
5
Perfectly
33
Q

Your patient has a C6 ASIA class A spinal cord injury which he sustained 8 weeks ago. He has been
noncompliant about attending therapy. Today he refuses to participate in therapy because he states
he has a headache. The nurses report poor urine output from the Foley catheter in the last 3 hours.
You order
(a) intravenous bolus of normal saline.
(b) push oral fluids and go to therapy.
(c) replacement of catheter.
(d) visit from peer mentor.

A

c) Autonomic dysreflexia must be ruled out. A Foley kink, or plugged catheter can distend the
bladder, causing autonomic dysreflexia with headache (and, ultimately, hypertension, piloerection
and flushing). The catheter should be checked for twists and kinks and be flushed. If urine/flush
return is poor, the catheter should be changed.

How well did you know this?
1
Not at all
2
3
4
5
Perfectly
34
Q

A patient with squamous cell carcinoma of the larynx elected to undergo organ preservation therapy
with intensive external beam radiation therapy to the anterior neck. He now presents with painless
bilateral lower extremity weakness. Which initial diagnostic test is most likely to be abnormal?
(a) Magnetic resonance imaging of the brain
(b) Nerve conduction studies of the lower extremities
(c) Thyroid stimulating hormone level test
(d) Computed tomography scan of the abdomen

A

(c) Patients who receive external beam radiation therapy to the anterior neck are at risk for developing
hypothyroidism. If their thyroid function tests are not monitored, they may initially present with
signs and symptoms of hypothyroidism. Myopathy is a common presenting complaint. This patient
is also at risk for radiation induced cervical myelopathy. However, since radiation
hypofractionation techniques have become the standard of care, the incidence of this dreaded
complication has significantly diminished.

How well did you know this?
1
Not at all
2
3
4
5
Perfectly
35
Q
Which measure is the first sign of respiratory muscle dysfunction in boys with Duchenne muscular
dystrophy?
(a) Vital capacity
(b) Oxygen saturation
(c) Maximal expiratory force
(d) Negative inspiratory force
A

(c) Recent studies by McDonald and by Bach showed that reduction of maximal expiratory force
(MEF) to 40%–60% of normal in the 7- to 14-year-old age group was the first sign of respiratory
muscle dysfunction in boys with Duchenne muscular dystrophy (DMD). The earlier and more
severe decreases of MEF that are greater than the decreases in maximal inspiratory force,
correspond to the clinically observed weakness of abdominal muscles, which like coughing are
important in forced expiration. Vital capacity was not found to decrease until an average of 15–16
years. Low oxygen saturation is a late manifestation in DMD, developing after hypercapnia.

How well did you know this?
1
Not at all
2
3
4
5
Perfectly
36
Q

In which circumstance is supracondylar suspension on a transtibial prosthetic socket most indicated?

(a) A 4-centimeter residual limb length below the tibial tubercule
(b) A residual limb with mildly adherent distal scar tissue
(c) A cylindrical-shaped residual limb
(d) A residual limb with poor definition above the femoral condyles

A

(a) Supracondylar suspension would be most indicated for an individual with a short transtibial residual
limb to provide additional mediolateral support and to increase the weight-bearing surface area for
more even pressure distribution. A cylindrical-shaped transtibial limb is ideal for allowing total
contact between the residual limb and the socket and is not an indication for supracondylar
suspension. Supracondylar suspension is also not specifically indicated for mildly adherent scar
tissue. Supracondylar suspension would be difficult to utilize in a residual limb that has poor
definition above the femoral condyles

How well did you know this?
1
Not at all
2
3
4
5
Perfectly
37
Q

A 14-year-old soccer player seen on the day of injury is unable to bear weight on her right foot. On
examination, she has significant swelling and mild ecchymosis laterally. The patient is tender over
the lateral side of her ankle. X-rays are negative. Your initial plan of management should include
(a) a walking cast for 4–6 weeks.
(b) nonsteroidal anti-inflammatory drugs and contrast baths for 48 hours.
(c) an air stirrup for 2 weeks with weight bearing as tolerated.
(d) an ankle brace and non-weight bearing for 3–4 weeks.

A

(c) This patient has suffered a fairly significant ankle inversion sprain. The lateral collateral ligaments
are injured 85% of the time with this type of injury. This patient should be placed in an air stirrup
for 2 weeks, weight bearing as tolerated, using crutches as needed. A cast or cast boot is acceptable
for 2 weeks but usually is not required. Contrast baths are not indicated before 48 hours following
injury.

How well did you know this?
1
Not at all
2
3
4
5
Perfectly
38
Q

(This question has been eliminated from the exam, therefore, it was not scored.)
On a pharmacologic basis, which agent used to decrease gastric acid secretion is most appropriate
after brain injury?
(a) Ranitidine
(b) Famotidine
(c) Omeprazole
(d) Sucralfate

A

(c) (This question has been eliminated from the exam, therefore, it was not scored.)
Choices a and b are both histamine type-2 (H2) blockers that are cognitively impairing

How well did you know this?
1
Not at all
2
3
4
5
Perfectly
39
Q

Which of the following is NOT associated with a Charcot joint?

(a) Diabetes
(b) Syringomyelia
(c) Tabes dorsalis
(d) Rheumatoid arthritis

A

(d) Destruction of a joint due to loss of nociceptive input describes a Charcot joint.

How well did you know this?
1
Not at all
2
3
4
5
Perfectly
40
Q

A 17-year-old person presents with frequent falls and no other complaints. On physical examination
you note unusually high arches, distal lower limb strength of 4/5, and decreased reflexes. The most
appropriate next step in the diagnosis would be
(a) nerve conduction studies.
(b) muscle biopsy.
(c) magnetic resonance imaging of the lumbosacral spine.
(d) serum heavy metal levels.

A

(a) The clinical picture is suggestive of a polyneuropathy. Patients with hereditary polyneuropathies
may have feet with high arches. Electrodiagnostic studies including nerve conduction studies
remain the most important first tests in the evaluation of polyneuropathy

How well did you know this?
1
Not at all
2
3
4
5
Perfectly
41
Q

Regarding the American Spinal Injury Association (ASIA) classification in prognosticating recovery,
(a) ASIA class A has a reasonable probability of improvement if there is no concurrent brain
injury.
(b) preservation of pinprick in ASIA class B carries a better potential for ambulation than
preservation of light touch sensation.
(c) recovery statistics for ASIA class C do not include the central-cord syndrome.
(d) Brown-Séquard’s syndrome has the worst potential for ambulation in ASIA class D.

A

(b) The presence of sensation in the sacral (S3–S5) dermatomes in patients with motor complete injury
indicates a favorable prognosis in terms of motor recovery, with pinprick sparing having the closest
correlation for motor recovery. Motor segments in the zone of injury in patients with complete
injury and an initial strength of 0/5 were more likely to recovery strength of 3/5 or more at 1 year if
the sensation in the corresponding dermatomes was intact. Most patients originally categorized as
ASIA (or Frankel) class A who progressed to ASIA class D or E had sustained traumatic brain
injury with cognitive impairment and were incorrectly diagnosed initially as class A.

How well did you know this?
1
Not at all
2
3
4
5
Perfectly
42
Q

A patient with far advanced prostate cancer metastatic to liver and bone achieves pain control
through the use of a subcutaneous hydromorphone infusion. The therapy is complicated by
significant sedation. You therefore initiate therapy with
(a) Clonidine (Catapres).
(b) Pemoline (Cylert).
(c) Methylphenidate (Ritalin).
(d) Naloxone (Narcan).

A

(c) Opioid-based pharmacotherapy is the current standard of care for severe cancer-related pain. Side
effect management is an integral dimension of competent pain management. Opioid-induced
sedation can be managed through the use of psychostimulants such as methylphenidate (Ritalin).
Epidural and intrathecal drug delivery can also be used to minimize neuropsychological toxicity.
Pemoline (Cylert), a psychostimulant, is not widely used because of concern over hepatotoxocity,
particularly in patients with liver metastases.

How well did you know this?
1
Not at all
2
3
4
5
Perfectly
43
Q

A functional capacity evaluation is required when

(a) the injured worker is ready to return to his/her job.
(b) the worker’s ability to perform work-related activities must be assessed.
(c) an injured worker’s case remains open more than 7 weeks.
(d) the company physician must determine whether the worker is injured.

A

(b) A functional capacity evaluation (FCE) is an assessment of a worker’s ability to perform workrelated
activities. A functional capacity examination can be used to determine if a worker might
benefit from work hardening or work conditioning, to determine whether a worker can return to
his/her job, to determine if work restrictions are recommended or if job modifications are needed,
and to document the worker’s activity capability. No absolute time line exists. An FCE can be used
in the subacute, or maintenance phase of treatment. An FCE does not determine validity of injury
but can reveal the effort a person expends to perform a task.

How well did you know this?
1
Not at all
2
3
4
5
Perfectly
44
Q

A 10-year-old child with L4-5 myelodysplasia and shunted hydrocephalus develops spasticity in her

legs. The most likely cause of this spasticity is
(a) shunt malfunction.
(b) symptomatic Chiari malformation.
(c) growth.
(d) tethered cord.

A

(d) Tethered cord is the most common cause of new onset spasticity in patients with myelodysplasia.
Linear growth does not cause new spasticity. Symptoms of Chiari malformation include cranial
nerve disorders and respiratory problems. Shunt malfunction may be associated with headaches,
vomiting, eye muscle abnormalities, and sometimes abdominal symptoms.

How well did you know this?
1
Not at all
2
3
4
5
Perfectly
45
Q

What is the shortest functional level for a transtibial amputation?

(a) Just proximal to the tibial tuberosity
(b) Just distal to the tibial tuberosity
(c) Six centimeters distal to the tibial tuberosity
(d) Ten centimeters distal to the tibial tuberosity

A

(b) The shortest functional amputation level for a transtibial amputation is just distal to the tibial
tuberosity. Knee flexion and extension can occur with this level of amputation because the patella
tendon and hamstring tendon attachments are still present. Control of knee flexion and extension of
the knee is lost with amputations proximal to the tibial tuberosity.

How well did you know this?
1
Not at all
2
3
4
5
Perfectly
46
Q

Which muscle does NOT depress the scapula?

(a) Serratus anterior
(b) Rhomboid major
(c) Latissimus dorsi
(d) Pectoralis minor

A

(b) The rhomboid major elevates, retracts and causes medial (downward) rotation of the scapula, but it
does not depress the scapula. All the other muscles depress the scapula.

How well did you know this?
1
Not at all
2
3
4
5
Perfectly
47
Q

Six months after a moderate traumatic brain injury, a 32-year-old woman complains of daytime
somnolence. Her medical work-up is negative. She has normal sleep patterns. The medication you
would most likely consider in this case is
(a) donepezil.
(b) buspirone.
(c) tolcapone.
(d) modafinil.

A

(d) From the information given, it is clear that this patient is functioning well overall. She has some
difficulty staying awake. Of the answers given, modafinil is the medication most appropriate to
help with alertness during the day. Donepezil is an acetylcholinesterase inhibitor used most often to
improve memory. Buspirone is used to decrease anxiety. Tolcapone is a newer dopaminergic agent
that has not been studied in populations with brain injury.

How well did you know this?
1
Not at all
2
3
4
5
Perfectly
48
Q

Which of the following may be associated with a subacromial corticosteroid injection?

(a) Dermal keratinification
(b) Localized osteopenia
(c) Tendon rupture
(d) Dermal hyperpigmentation

A

(c) Intra-articular corticosteroid injections have enough systemic absorption that suppression of the
adrenal hypopituitary axis may be seen with repeated injections. More localized deleterious effects
include skin depigmentation, soft tissue atrophy, steroid arthropathy, postinjection flare, and tendon
rupture.

How well did you know this?
1
Not at all
2
3
4
5
Perfectly
49
Q

What advantage of a reciprocating gait orthosis applies for a person with T10 ASIA class A spinal
cord injury?
(a) The wearer ambulates with a relatively stable 4-point gait.
(b) It accommodates adductor spasticity through the cable mechanism.
(c) It allows longer steps than with traditional knee-ankle-foot orthoses.
(d) Gait is smooth because the pelvis is never stationary during the gait cycle.

A

(a) The advantages of the reciprocating gait orthosis (RGO) include enabling the wearer to ambulate
with a relatively stable 4-point or 2-point crutch gait. The cable mechanism prevents the patient
from taking an unduly long step that would make balance recovery difficult. The torso is stabilized
with an orthosis, which may also benefit patients who have thoracic spinal cord injury. Ambulation
with RGO is associated with a lower pulse rate and a lower heart rate than with other trunk-hipknee-
ankle-foot ortheses (THKAFOs). Velocity of the ambulation is somewhat faster and patients
appear to be able to manage longer distances than with other THKAFOs. The disadvantage of the
RGO is that donning is time consuming. The gait is slow and is not smooth because the pelvis is
stationary momentarily during each gait cycle. The orthosis is heavy. Marked spasticity of the
adductors disturbs the operation of the cable mechanism.

How well did you know this?
1
Not at all
2
3
4
5
Perfectly
50
Q

An elderly patient with breast cancer and widespread osseous metastases has developed acute onset,
severe low thoracic back pain while stooping forward to lift groceries. In addition to formulating a
pain management program and initiating bisphosphonate therapy, what brace do you prescribe for
this patient?
(a) Jewett
(b) Knight-Taylor
(c) Williams
(d) Chairback

A

(a) The patient has developed a vertebral compression fracture. An extension brace will minimize her
pain and possibly reduce further pathological compression fractures. The Knight-Taylor brace
provides thoracolumbar spine control in the saggital and coronal planes. The Williams brace is a
lumbosacral extension-lateral control orthotic. The chairback brace is an example of a lumbosacral
flexion-extension control orthosis.

How well did you know this?
1
Not at all
2
3
4
5
Perfectly
51
Q

How does work conditioning differ from work hardening? Work conditioning is

(a) a maintenance exercise program.
(b) activities simulating the worker’s tasks.
(c) training the worker for a specific job.
(d) aerobic training that may not be job specific.

A

(d) Work conditioning is the physical conditioning portion of work hardening. It has been referred to as
an aerobic training program for patients with less complex and more chronic conditions. Often
these patients are not being retrained to return to a specific job. In a work hardening program the
injured worker is gradually strengthened and reconditioned to the functional capacity level required
to perform a given job. Work hardening has also been referred to as work simulation, work
readiness training, and work rehabilitation

How well did you know this?
1
Not at all
2
3
4
5
Perfectly
52
Q

What is the earliest electrophysiologic abnormality seen in generalized myasthenia gravis?
(a) Increased jitter on single fiber electromyography of the extensor digitorum communis.
(b) Blocking on single fiber electromyography of the extensor digitorum communis.
(c) A 10% decrement of compound motor action potential (CMAP) amplitude with 2–3Hz
repetitive stimulation, recording from the abductor digiti minimi.
(d) A 10% decrement of CMAP amplitude with 2–3Hz repetitive stimulation, recording from the
frontalis.

A

(a) Increased jitter on single-fiber electromyography is the earliest abnormality seen in myasthenia
gravis. The other abnormalities noted are seen later in the disease process.

How well did you know this?
1
Not at all
2
3
4
5
Perfectly
53
Q

A 9-year-old girl with an L1 ASIA class A spinal cord injury that occurred at age 5 years presents in
your office with a 1-day history of a swollen left leg. History is that she woke up with the swollen
leg the day before. There is no history of trauma, fever, or shortness of breath. On examination, you
find a prepubertal girl in no distress with normal vital signs. Upper extremities are normal. Lower
extremities have moderate spasticity and no voluntary movement. Skin is normal. The left leg is
warm and swollen from the ankle to the knee. There is no sensation in the legs. Which test is most
likely to yield the correct diagnosis?
(a) Bone scan
(b) Plain radiograph
(c) Venous Doppler study
(d) White blood cell count with differential

A

(b) Deep venous thromboses (DVTs) which can be diagnosed by Doppler study usually occur in the
first 3 months after spinal cord injury (SCI) and are rare in prepubertal children. In lower leg DVTs
the foot and leg are usually swollen. Heterotopic ossification (HO), which can be detected by bone
scan, occurs in about 3% of children with SCI and has onset an average of 14 months after injury.
Heterotopic ossification most commonly involves the hip. Cellulitis is usually associated with skin
lesions and usually involves a discrete area. A fracture is the most likely cause of swelling in this
case and can be diagnosed by plain radiographs.

How well did you know this?
1
Not at all
2
3
4
5
Perfectly
54
Q

What is the best-established benefit of a microprocessor-controlled knee unit, compared to a
conventional pneumatic knee unit?
(a) It provides both swing and stance phase control.
(b) It allows greater knee flexion to perform bending and lifting activities.
(c) It allows running at faster speeds.
(d) It provides a more natural and symmetric gait pattern.

A

(d) Microprocessor controlled knee units use a computerized system to automatically adjust the knee
unit’s resistance over a wide range of gait speeds. With this automatic control, there is greater
consistency and reliability in the knee movement during the gait cycle. These benefits give the
amputee greater confidence and improve swing phase responsiveness and gait symmetry, as well.
Negative aspects of the microprocessor controlled knee units are heavier weight and greater
expense. These units have not been shown to improve running speed.

How well did you know this?
1
Not at all
2
3
4
5
Perfectly
55
Q

A 45-year-old man presents to his physician with complaints of posterior heel pain for several weeks.
The pain is worse when he first begins to ambulate after resting. He has a noticeable limp. He denies
any recent trauma. On examination, he is exquisitely tender along the medial and lateral aspects of
the calcaneus and along the Achilles tendon at its most distal portion. The area is mildly swollen and
warm to touch. What is the most likely diagnosis?
(a) Achilles tendon rupture
(b) Plantar fasciitis
(c) Posterior calcaneal nerve entrapment
(d) Retrocalcaneal bursitis

A

(d) This scenario is most likely retrocalcaneal bursitis. This condition is seen in middle aged to elderly
persons, and may occur with overuse. A limp is common and pain is usually worse with first
activity after rest. Swelling and local tenderness may be seen, and a “pump bump” may be present.
This bump is a prominence created by inflammation associated with shoe wear. The local swelling
is due to inflammation of the retrocalcaneal bursa. An acute Achilles tendon rupture is associated
with a positive Thompson test, which consists of compressing the calf and causing the foot to
plantarflex. If the tendon is ruptured, the foot will not plantarflex. The pain is not located in the
bottom of the foot as would be seen with plantar fasciitis. Posterior calcaneal nerve entrapment
would not cause swelling, nor would it be warm to touch.

How well did you know this?
1
Not at all
2
3
4
5
Perfectly
56
Q
Which is the best predictor of discharge from a rehabilitation center to home for a man who has had
a stroke?
(a) Lesion location
(b) Shoulder pain
(c) Ambulatory status
(d) Bladder incontinence
A

(d) Of the choices presented above, the most consistent predictor of good outcome and discharge home
is bladder continence. Probably the strongest overall predictor of ability is admission functional
ability (which reflects severity of stroke).

How well did you know this?
1
Not at all
2
3
4
5
Perfectly
57
Q

A 23-year-old man complains of localized low back pain that is worse in the morning and improves
as the day progresses with activities. He has no radicular symptoms and a nonfocal neurologic
examination. Which finding unequivocally supports your diagnosis
(a) A positive human leukocyte antigen B27
(b) Bilateral sacroiliitis on a radiograph of the pelvis
(c) An elevated erythrocyte sedimentation rate
(d) Atlantoaxial subluxation on a radiograph of the cervical spine

A

(b) Bilateral spondylitis is diagnostic of ankylosing spondylitis. HLA-B27 is not helpful because a
percentage of the normal population is positive for HLA-B27 and not all patients with ankylosing
spondylitis are positive. Erythrocyte sedimentation rate is a general marker of inflammation.
Enthesopathies may occur in a wide variety of inflammatory disorders, including
spondyloarthropathies.

How well did you know this?
1
Not at all
2
3
4
5
Perfectly
58
Q

For a patient with myopathy, which type of exercise is most harmful?

(a) High intensity concentric
(b) High velocity isokinetic
(c) High intensity eccentric
(d) High intensity isotonic

A

(c) There is evidence of more muscle fiber damage with eccentric exercise. This appears to be true for
persons with myopathic disease as well as in control subjects. There is similar evidence in animal
studies. The type of exercise and its potential benefits versus detrimental effects continues to be a
controversial subject. However, more and more evidence suggests that exercise in persons with
myopathy may not be as detrimental as once thought.

How well did you know this?
1
Not at all
2
3
4
5
Perfectly
59
Q

A 52-year-old woman is seen for follow-up 1 year after right carpal tunnel release. She had good
initial relief of her symptoms following the release, but has had a 4-month history of recurrent symptoms in the right wrist and hand. Electromyography of the right upper extremity and cervical paraspinals is normal. Her nerve conduction studies (with normal values in parentheses) are as follows:
Motor Nerve Conduction
Nerve Segment Distal Latency (ms) Amplitude (mV) Velocity (m/s)
R. Median forearm 4.5 (5) 50 (>45)
R. Ulnar forearm 3.4 (5) 52 (>45)
R. Ulnar across elbow 9.5 55
L. Median forearm 3.5 10.0 (>5) 54 (>45)

Sensory
Nerve Segment Peak Latency (ms) Amplitude (uV)
R. Median 14cm antidromic-digit II 3.9 (10)
R. Median 7cm antidromic-digit II 1.9 15
R. Ulnar 14cm antidromic-digit V 3.2 (10)

You conclude that the patient has

(a) compression of the median nerve at Guyon’s canal.
(b) pronator syndrome.
(c) entrapment of the median nerve at the ligament of Struthers.
(d) normal postoperative findings.

A

(d) After a successful carpal tunnel release median distal latencies improve, but often do not return to
normal. Mild residual slowing is not unusual

How well did you know this?
1
Not at all
2
3
4
5
Perfectly
60
Q

Which positive effect of ankle-foot orthotics has been proven beneficial in the treatment of children
with cerebral palsy?
(a) Improved gait efficiency as measured by gait analysis
(b) Prevention of contractures
(c) Improved knee extensor strength
(d) Decreased plantar flexor posture

A

(a) There are no large, randomized, controlled studies that show the long-term effects of any type of
Ankle-Foot Orthosis (AFO) on function or contracture formation. Small studies have shown that
both rigid and hinged AFOs improve gait efficiency by preventing plantar flexion.

How well did you know this?
1
Not at all
2
3
4
5
Perfectly
61
Q

Regarding energy expenditure during prosthetic ambulation,
(a) at self selected walking speeds, someone ambulating with a transtibial prosthesis has a higher
rate of metabolic energy expenditure [mlO2/(kg•meter)] compared with normal human
locomotion.
(b) whether a person has a transtibial or transfemoral prosthesis, the metabolic cost
[mlO2/(kg•meter)] of ambulation is the same.
(c) at self selected walking speeds, cardiac work load during transtibial prosthetic ambulation is
comparable to that in normal human locomotion.
(d) gait velocity [meters/min] during transtibial prosthetic ambulation is comparable to that in
transfemoral prosthetic ambulation.

A

(c) The rate of metabolic energy expenditure–mlO2/(kg•minute)–represents energy expenditure per unit
of time in comparison to metabolic cost–mlO2/(kg•minute)–which is a measure of energy
expenditure per unit distance, a meter. The metabolic cost of prosthetic ambulation is greater than
that used in normal human locomotion and the metabolic cost of transfemoral prosthetic ambulation
is greater than that in transtibial prosthetic ambulation. Because individuals who ambulate witha
prosthesis slow their gait velocity, their rate of metabolic energy expenditure is relatively
unchanged compared to normal ambulation at self-selected walking speeds. Because the rate of
metabolic energy expenditure is maintained at a level comparable to ambulation without a
prosthesis, cardiac work load is also unchanged during ambulation with either a transtibial or
transfemoral prosthesis, if subjects are allowed to walk at their self-selected walking speed.

62
Q

Which statement regarding Waddell signs is TRUE?

(a) They are a collection of organic signs that identify malingerers.
(b) They include simulation, distraction, overreaction, and regional disturbances.
(c) A finding of 2 or more positive signs is clinically significant.
(d) These signs are seen much more commonly in chronic pain patients.

A

(b) The Waddell signs are nonorganic physical signs that are used for patients with low back pain.
They are a simple screen that can be used to help identify patients who may require a more detailed
psychological assessment. Three or more positive signs are a clinically significant finding. The
signs include simulation, distraction, overreaction, regional disturbances and either superficial or
nonanatomic tenderness.

63
Q

A 26-year-old man was in a motor vehicle crash last night. Oxygen saturation was consistently
around 93%. Intracranial pressure was 15mmHg, with a mean arterial blood pressure of 110mmHg.
Pupils were equally reactive. This patient’s prognosis is
(a) poor because oxygen saturation was consistently below 95%.
(b) poor because of the high intracranial pressure.
(c) not affected by pupillary response in the first 24 hours.
(d) good because his cerebral perfusion pressure was greater than 70mmHg

A

(d) The guideline set by the American Association of Neurological Surgeons included avoidance of
oxygen saturations of less than 90%. Cerebral perfusion pressure is a more important predictor of
outcome than ICP since CPP is more directly related to metabolic delivery and ischemia. CPP is
mean arterial pressure minus ICP and in this patient is 95. Pupillary response is a very important
early predictor of eventual outcome.

64
Q

Regarding the mechanism of nonsteroidal anti-inflammatory medications, which statement is
TRUE?
(a) They are required for the synthesis of prostaglandins.
(b) Cyclooxygenase 2 protects the gastric mucosa.
(c) Cyclooxygenase 2 is primarily induced by inflammatory stimuli.
(d) They inhibit synthesis of arachidonic acid.

A

(c) Nonsteroidal anti-inflammatory medications inhibit the cyclooxygenase enzyme (COX) which is
necessary for prostaglandin and thromboxane synthesis. At least two isoforms of cyclooxygenase
have been identified, named COX 1 and COX 2. COX 1 is in the gastric mucosa where the
prostaglandin synthesis has a protective effect; COX 2 is primarily induced by inflammatory stimuli
and is the target of the new COX 2 inhibitors, which have less gastric toxicity.

65
Q

A 47-year-old left handed assembly line worker presents with a 1-week history of neck pain and left
arm weakness. Examination reveals diminished left triceps reflex. Muscle strength is 4/5 left elbow
extensor, 4-/5 wrist extensor, and otherwise 5/5. Sensation is decreased to pinprick in the left index
and middle fingers. What is the most appropriate initial intervention?
(a) Refer to a neurosurgeon
(b) Start on a tapering steroid
(c) Start cervical extension exercises
(d) Apply therapeutic ultrasound

A

(b) The clinical presentation is that of a person with an acute cervical radiculopathy. In this setting the
use of steroids is often indicated for a potent anti-inflammatory effect. The treatment is typically
initiated when an inflammatory etiology is suspected and there are no contraindications such as an
infection or history of peptic ulcer disease.

66
Q

A 28-year-old with T11 paraplegia for 6 months comes to your office to discuss treatment options
for erectile dysfunction. A trial of sildenafil (Viagra) was unsuccessful. He asks about constriction
rings. You tell him
(a) they should not be used in men with sickle cell disease.
(b) they can be kept in place for up to 2 hours.
(c) they cannot be used without a vacuum pump.
(d) they are not effective for erectile dysfunction in spinal cord injury.

A

(a) Constriction rings (cock rings) to occlude venous outflow can be used if a person is having a poorly
sustained erection. A vacuum pump and constriction rings can be used if a person is having no
erections. To prevent skin breakdown from prolonged venous congestion within the penis, the ring
should not be kept in place for more than 30 minutes. They should not be used in men with sickle
cell disease. Anticoagulants are a relative contraindication to their use.

67
Q

A young breast cancer survivor expresses concern over her risk of developing lymphedema. You
explain that an increased incidence of lymphedema is associated with
(a) hypertension.
(b) diabetes mellitus.
(c) osteoarthritis.
(d) obesity.

A

(d) Obesity and weight gain has consistently been shown to increase breast cancer survivors’ risk of
developing lymphedema.

68
Q

What is the rationale for bracing for low back pain in the injured worker?

(a) It has been shown to prevent further low back pain injury.
(b) It prevents further injury and should be used in all injuries occurring at work.
(c) It has been shown to improve lifting capabilities.
(d) It can provide proprioceptive feedback to reinforce proper mechanics.

A

(d) Lumbar supports do not prophylactically prevent low back injuries or prevent recurrence of low
back pain. One study showed a decrease in lost time at work when lumbar supports were utilized
with a back injury educational program. No improvement in lifting capacity has been found with
the use of bracing. Workers who use lumbar supports use them as a proprioceptive reminder to use
proper mechanics with lifting and other work-related activities.

69
Q

Which potentials have manifestations that CANNOT be observed on physical examination?

(a) Fasciculation potentials
(b) Myokymic discharges
(c) Complex repetitive discharges
(d) Cramp potentials

A

(c) Complex repetitive discharges can only be detected with electromyography. The remainder of the
discharges have manifestations that can be observed on physical examination.

70
Q

Which circumstance is most likely to result in left knee hyperextension at initial contact (heel strike)
in a patient with spastic left hemiparesis?
(a) Solid ankle, double-metal, upright ankle-foot orthosis set in 5° of plantar flexion
(b) Addition of a solid one-quarter inch heel to the left shoe
(c) Addition of ankle dorsiflexion spring assistance to a dual-channel, double-metal, ankle-foot
orthosis
(d) Addition of a T-bar strap to a double-metal upright ankle-foot orthosis to reduce foot inversion

A

(a) Placing a solid ankle, double-metal, upright ankle-foot orthosis in 5° of plantarflexion will increase
the extension moment (forces) at the knee and this can result in knee hyperextension, especially at
the time of initial contact (heel strike). Addition of a T-bar strap should not effect the knee and the
addition of dorsiflexion spring assistance or a one-quarter inch heel will reduce the tendency to
have knee hyperextension.

71
Q

Regarding anterior shoulder dislocation, which statement is the most correct?

(a) The most common neural injury is a lower trunk brachial plexus injury.
(b) Return to full contact sports is allowed only after isokinetic values are 100% of normal.
(c) To prevent capsular adhesions, Codman exercises are allowed starting 48 hours postinjury.
(d) Initial rehabilitation (1–3 weeks) should avoid active external rotation past 45°.

A

(d) Following anterior shoulder dislocation, the most common neural injury is damage to the axillary
nerve. Gentle pendular exercises (Codman exercises) are not allowed until weeks 2–3 postinjury.
Early postreduction rehabilitation should avoid active external rotation past 45° for at least 3 weeks
and usually up to 6 weeks. External rotation with abduction is not allowed until 8 weeks postinjury.
Return to contact sports is not allowed until isokinetic values reach 85% of normal.

72
Q

A patient with focal right upper extremity spasticity initially demonstrated a good response to
electromyographically-guided botulinum toxin injections, as measured functionally and on the
Modified Ashworth Scale. He received 2 sets of follow-up injections, each 6 months apart, when
spasticity returned. He returns 4 weeks after his most recent injection, complaining that he has not
seen any effect. Of the options given, what is the most likely explanation for this lack of effect?
(a) Diffusion characteristics of botulinum toxin change with repeated administration.
(b) It becomes very difficult to localize spastic muscles with repeated administration.
(c) Antibodies have developed to botulinum toxin, neutralizing it.
(d) The hepatic enzymes that metabolize botulinum toxin have increased with repeated
administration.

A

(c) Diffusion characteristics do not change. Electromyographic guidance helps to ensure that the
botulinum toxin is injected into the targeted muscle. Localization does not become more difficult
with repeated injections when spasticity returns. Botulinum toxin is not metabolized by the liver.
The development of neutralizing antibodies is the primary reason for loss of efficacy with repeated
injections.

73
Q

(This question has been eliminated from the exam, therefore, it was not scored.)
A 45-year-old man presents with diplopia and ataxia 1 week after a viral infection. Physical
examination reveals areflexia and ophthalmoplegia. Electrodiagnostic evaluation demonstrates
reduced sensory nerve action potential amplitudes. Which course is expected in this disorder?
(a) Need for ventilatory support
(b) Muscle pain
(c) Complete resolution
(d) Improvement with plasma exchange

A

(c) (This question has been eliminated from the exam, therefore, it was not scored.)
The Miller-Fisher variant of Guillain-Barré syndrome consists principally of a triad of
ophthalmoplegia, ataxia, and areflexia. Altered sensation in the hands and around the mouth can
also occur. Most patients first note the sudden onset of diplopia. This is generally a benign
condition and does not require specific immune therapy. Pure sensory and pure motor are other
forms of Guillain-Barré syndrome.

74
Q

What information does a functional capacity evaluation provide the treating physician?
(a) It indicates the worker’s ability to return to full duty or return to work with restrictions.
(b) It shows the worker’s maximum oxygen uptake while simulating work related activities.
(c) It provides standardized test results that can be compared with national normative data.
(d) It differentiates discriminates between psychiatric illness and behavioral symptom
magnification.

A

(a) The information provided by functional capacity evaluations (FCE) includes the worker’s capability
to return to full or restricted duty, the need for work hardening, or the need for work conditioning.
It can also define discrepancies between subjective complaints and objective findings. The FCE can
also help determine if the worker has reached maximal medical improvement. Although general
aerobic condition may be assessed, the FCE does not include formal aerobic capacity testing. The
psychological assessment is important during the FCE but formal psychiatric diagnosis is not made
during an FCE. Unfortunately, functional capacity evaluations are not standardized and cannot be
compared. Such standardization has been proposed but currently does not exist.

75
Q

Which statement regarding myotonic discharges is TRUE?

(a) They are variable in frequency.
(b) They have a constant amplitude.
(c) They arise from the anterior horn cell.
(d) They are rarely seen without clinical myotonia.

A

(a) Myotonic discharges consist of repetitive waveforms of similar configuration which wax and wane
in their frequency and amplitude.

76
Q

Juvenile rheumatoid arthritis (JRA) differs from adult onset rheumatoid arthritis: in JRA

(a) joint destruction occurs earlier.
(b) large joint involvement is less frequent.
(c) the cervical spine is involved less frequently.
(d) systemic features are more common.

A

(d) Children with juvenile rheumatoid arthritis are more likely to have systemic features, have large
joints involved, and have cervical spine involvement. Adults with rheumatoid arthritis have joint
destruction earlier.

77
Q

Which spinal orthosis is used to prevent thoracic spinal flexion by providing 3-point pressure over
the sternum and pubis anteriorly and the upper lumbar spine posteriorly?
(a) Custom molded, plastic thoracolumbosacral orthosis
(b) Lumbosacral corset with posterior metal stays
(c) Jewett orthosis
(d) Taylor orthosis

A

(c) Several different types of thoracolumbosacral (TLSO) orthoses are available to control segmental
spine motion in this region. A custom molded plastic TLSO provides almost total contact support
for uniform pressure distribution and control of motion in all plains. A lumbosacral corset with
metal stays provides support circumferentially and helps reduce spine motion primarily in the
lumbosacral region. A Taylor orthosis also provides circumferential support with the addition of
axillary straps. The Taylor orthosis is primarily designed to resist flexion and extension. A Jewett
brace is designed to limit thoracic spine flexion by providing 3-point pressure over the sterum and
pubis anteriorally and the upper lumbar spine posteriorally. This type of brace is used most
commonly for individuals with thoracic spine anterior compression fractures.

78
Q

Which of the following is an appropriate compensatory technique for managing dysphagia?

(a) Tilting the head to the weaker side
(b) Glossopharyngeal breathing
(c) Chin tuck
(d) Turning the head to the stronger side

A

(c) Tilting the head to the stronger side and turning the head to the weaker side (but not vice versa) are
appropriate compensatory techniques. Glossopharyngeal breathing is used in pulmonary
rehabilitation and has no value in dysphagia management.

79
Q

A 70-year-old man comes to your office complaining of posterior neck and bilateral shoulder pain
worse in the morning. Radiographs of his shoulders reveal glenohumeral osteoarthritis, and his
erythrocyte sedimentation rate is 70. Your initial management is to
(a) perform bilateral glenohumeral corticosteroid injections.
(b) prescribe Codman exercises.
(c) obtain a cervical spine magnetic resonance image.
(d) prescribe prednisone 20mg per day.

A

(d) This patient has polymyalgia rheumatica. When you diagnose polymyalgia rheumatica the patient
is placed on daily prednisone and should note dramatic improvement within a few days.

80
Q

Muscle function changes after polio consist of all of the following EXCEPT

(a) muscle fiber hypertrophy.
(b) increase in motor unit size.
(c) decreased jitter on single fiber electromyography.
(d) increased fiber density on single fiber electromyography.

A

(c) Polio affects the anterior horn cells, resulting in motor axon dropout. Compensatory effects include
individual muscle fiber hypertrophy of the surviving muscle fibers and increase in motor unit size
due to sprouting and innervation of some of the surrounding denervated muscles. There is also an
increase in fiber density as a result of re-innervation as well as atrophy and replacement of
denervated muscles. This can be assessed by single fiber electromyography. Jitter also can be
increased because of the re-innervated muscle fibers’ instability.

81
Q
What is the most common level of occult spine fracture after trauma that is missed by plain
radiographs?
(a) C7/T1
(b) T5/T6
(c) T12/L1
(d) L4/L5
A

(a) Occult cervical fractures are most often seen at the C1 and C7 levels. By adding computed
tomography (CT) scanning to the evaluation of trauma patients, a significant number of occult
cervical fractures can be diagnosed. Of spinal fractures, 5%–30% are multiple and may appear at
noncontiguous levels. Thus, radiographic evaluation of the entire spinal axis is necessary whenever
injury at 1 region of the spine is detected.

82
Q

After cardiac rehabilitation, patients with coronary artery disease remain asymptomatic at exertional
levels that previously provoked angina. This change occurs primarily because of increased
(a) coronary collateralization and neoangiogenesis.
(b) serum hemoglobin concentrations.
(c) myocardial mitochondrial enzyme concentrations.
(d) oxidative enzyme concentrations in skeletal muscle.

A

(d) Peripheral training effects are largely responsible for the enhanced functional status of patients who
undergo cardiac rehabilitation. Increased oxygen extraction and a wider arteriovenous oxygen
difference have been described. Improved utilization of oxygen by active muscles, because of an
increase in oxidative enzymes, is an important factor.

83
Q

Cumulative trauma disorders

(a) develop as a result of repetitive macrotrauma.
(b) are most common in workers who work at a keyboard.
(c) occur after ergonomic recommendations have been implemented.
(d) result from repetitive microtrauma in the setting of poor ergonomics.

A

(d) Cumulative trauma disorders (CTDs) develop as a result of repetitive microtrauma to tissue in the
setting of poor ergonomics. Any worker who performs a task requiring repetitive loading or motion
is at risk. This includes keyboard operators but is not exclusive to them. The most common
occupational hazard has not been determined. Often, implementing ergonomic recommendations
can help reduce CTDs.

84
Q
Children with which physical disorder tend to have higher verbal skills compared to overall
cognitive ability?
(a) Muscular dystrophy
(b) Myelodysplasia
(c) Cerebral palsy
(d) Autism
A

(b) Children with myelodysplasia have deceptively good verbal facility that creates the impression of
higher intellectual functioning than is found on formal testing (“cocktail party syndrome”).
Children with cerebral palsy, autism, and muscular dystrophy do not typically demonstrate this
finding.

85
Q

For which individual would a high-profile, dynamic elastic response (energy-storing) prosthetic foot
and ankle mechanism be most indicated?
(a) 75-year-old man with a transtibial amputation who lives in a nursing home
(b) 53-year-old woman with a transfemoral amputation who is a recreational swimmer
(c) 60-year-old man with a transtibial amputation who enjoys jogging
(d) 43-year-old man with a transfemoral amputation who farms

A

(c) High-profile, dynamic elastic response prosthetic feet and ankle components such as the Flex-Foot
and Springlite are primarily indicated for individuals who are expected to be community ambulators
and are able to ambulate at variable cadences. This class of prosthetic components is also used for
individuals participating in running and sports activities. Therefore, this class of components would
be most indicated for the individual with a transtibial amputation interested in returning to running.
An individual who farms would likely benefit from a prosthetic foot and ankle system which is
more accommodative over uneven surfaces.

86
Q

A 5-year-old boy is brought to your office with pain in his left groin for the last 2 months. He has an
antalgic gait and favors his left leg. He has been unable to play with his siblings because of pain and
reports increased severity of the pain at night. There is no history of trauma. Passive range of motion
of the left hip is extremely painful. Which of the following is most likely?
(a) The child is not telling the truth about the trauma for fear of being punished by his parents.
(b) A simple radiograph will demonstrate this problem to be a benign syndrome that only requires
monitoring.
(c) This child is at risk for permanent disability and treatment requires the use of an abduction
brace.
(d) You are very concerned about physical abuse by the parents, since the child is very quiet when
the parents are in the examination room.

A

(c) This child has Legg-Calvé-Perthes disease. This is avascular necrosis of the femoral head (also
known as idiopathic osteonecrosis of the femoral head) and typically affects children between the
ages of 4 and 8 years. Boys are affected 4 times more than girls are, and it is unilateral in 90% of
children. It is uncommon in African American children. The bone dies and loses its structural
integrity, leading to collapse of the femoral head with deformity and arthritis. An abduction brace is
worn all day in an attempt to position the femoral head in the acetabulum. This will hopefully lead
to a more spherical head of the femur. The femoral head can revascularize and remodel, but this
occurs over several months. Osteotomy is usually reserved for older children.

87
Q

Which measure will ensure the best outcome for a 68-year-old man who is in the critical care unit
with severe traumatic brain injury?
(a) Minimizing cerebral perfusion pressure
(b) Minimizing early hypoxia
(c) Avoiding hypertension
(d) Inducing hypothermia

A

(b) Hypoxia in the setting of brain injury is associated with poor outcome. Maintaining perfusion
pressure and avoiding hypotension are important critical care measures to avoid secondary
complications in brain injury. Inducing hypothermia has not been found to improve outcome in
patients with brain injury.

88
Q

A 45-year-old woman with diabetes complains of shoulder pain and stiffness after a minor fall. On
examination you find severely restricted range of motion in all planes. There is no warmth, swelling,
tenderness to palpation, or deformity. Your treatment plan includes
(a) subacromial corticosteroid injection.
(b) an arm sling.
(c) McConnell taping.
(d) Codman exercises.

A

(d) The Codman pendulum exercises are recommended to treat adhesive capsulitis (frozen shoulder).

89
Q

Of the 4 disorders described below, which definitions is NOT correct?

(a) Hyperesthesia is an exaggeration of any sensory modality response.
(b) Allodynia is the experience of pain in response to a normally nonpainful stimulus.
(c) Paresthesia is a perversion of sensation, producing a perception that is abnormal in character.
(d) Hypesthesia is a diminution to painful stimuli.

A

(d) All the definitions are correct except for the definition of hypesthesia, which is not only diminution
to painful stimuli but also diminished sensation to any sensory modality. Paresthesias can also be
described as abnormal sensations in the absence of stimulation.

90
Q
In an individual with a C6 complete spinal cord injury, the ability to generate a “pinch” is produced
by
(a) wrist extension.
(b) elbow supination
(c) wrist flexion.
(d) elbow pronation.
A

(a) Wrist extension via extensor carpi radialis results in passive shortening of the (finger) flexor
tendons. This phenomenon is termed tenodesis.

91
Q

For optimal sensitivity, cardiac stress testing conducted 4 to 8 weeks following myocardial infarction
must proceed to what percentage of a patient’s age-adjusted maximal heart rate?
(a) 60%
(b) 75%
(c) 85%
(d) 90%

A

(c) For both functional and diagnostic testing, patients must proceed to 85% of their age-adjusted
maximal heart rate. Otherwise half of the abnormalities will be missed. Patients on beta blockers
should be tested to a work load that would incur oxygen consumption of 80% of maximal oxygen
consuption (V02.max).

92
Q

Disablement as defined by the World Health Organization is
(a) all the effects of the injury or illness, impairments, activity limitations and barriers to
participation.
(b) inability to advance in the work place due to physical disability or need for workplace
adaptations.
(c) recognition that the work injury was the cause of the disability as determined by civil
litigation.
(d) restrictions in the work site that requires the worker to use special aids

A

(a) Disablement as defined by the World Health Organization is the term that summarizes all the
effects of the injury or illness, impairments, activity limitations, and barriers to participation
experienced by the individual. Workers receiving disability compensation have a disincentive to
return to work. Reforms have been made to allow workers to investigate their ability to return to
the work force without losing benefits. Disablement does not relate to advancement in the
workplace.

93
Q

A medical student has been practicing with the electromyography (EMG) machine. You note that
the conventional filter settings have been changed. The low frequency filter is now set at 200Hz and
the high frequency filter is set at 1,000Hz. This will cause
(a) a decreased common mode rejection ratio.
(b) increased input impedance.
(c) no significant changes.
(d) distortion of recorded potentials

A

(d) Typical settings are 20Hz for the low frequency filter and 10,000Hz for the high frequency filter.
Allowing such a narrow bandwidth will cause distortion of the recorded potentials. It would have
no effect on the input impedance or common mode rejection ratio.

94
Q
gambar ankle joint
A: tibionavicular joint
B: calcaneocuboid joint
C: talocalcaneal-small midfoot bones joint
D: metatarsal joint

At which location is a Syme amputation performed?

(a) A
(b) B
(c) C
(d) D

A

(a) A Syme amputation (A) is ankle disarticulation with tapering of the medial and lateral malleoli and
preservation of the heel pad for soft tissue coverage and distal weight bearing. A Boyd hindfoot (B)
amputation is a talectomy and calcaneal-tibial arthrodesis after forward translation of the calcaneus.
A Chopart level (C) amputation is performed at the mid-tarsal level with sparing of the talus and
calcaneus. A tarsal-metatarsal disarticulation (D) is also referred to as a Lisfranc amputation.

95
Q

A 60-year-old man comes into your office in a stooped forward position ambulating with a cane. He
is complaining of back pain, which he has had since his early twenties. On examination he is unable
to straighten up and appears to have a fused cervical spine, since his neck range of motion is almost
nonexistent. You suspect he has ankylosing spondylitis. Work-up will reveal which finding?
(a) Negative rheumatoid factor, normal erythrocyte sedimentation rate, negative antinuclear
antibody
(b) A normal Gillet test showing reasonably good sacroiliac motion
(c) Sparing of the thoracic spine evidenced by plain radiographs
(d) A negative result for human leukocyte antigen B27

A

(a) This patient has a seronegative spondyloarthropathy, ankylosing spondylitis. This usually begins in
the patient’s twenties and presents with pain in the low back and sacroiliac area. An advanced case
of this disease will show a patient with fused sacroiliac joints and ossification of the anterior
longitudinal ligament and posterior facets. This does not spare the thoracic spine but ascends from
the sacroiliac joints to the cervical spine, resulting in a C-curve of the spine. Ankylosing spondylitis
has a positive human leukocyte antigen B27 in 90% of patients and a negative rheumatoid factor,
negative antinuclear antibody and a normal erythrocyte sedimentation rate.

96
Q

Regarding central nervous system tumors in adults,

(a) meningiomas are the most common form of primary tumors.
(b) approximately 50% of these tumors are metastatic.
(c) glioblastoma multiforme has a median survival rate of 5 years.
(d) brain tumor treatment side effects do not affect outcome.

A

(b) Meningiomas are the second most common form of primary tumors. Gliomas are the most
common. Glioblastoma multiforme has a median survival rate of less than 1 year. Treatment side
effects do affect outcome. Metastatic disease comprises 50% of central nervous system tumors.

97
Q

An elderly person’s risk factors for falling include

(a) advanced age.
(b) living alone.
(c) osteoarthritis.
(d) polypharmacy.

A

(d) Risk factors for falling in the elderly include polypharmacy, particularly sedative use, cognitive
impairment, lower extremity disability, palmomental reflex, foot problems, peripheral neuropathy
and poor balance.

98
Q

A 25-year-old man presents with a 3-week history of left arm weakness after a fall from a
motorcycle. He has decreased pinprick sensation over the deltoid area, the lateral forearm, and the
thumb and index finger. Which test is most useful in prognostication?
(a) Electrodiagnostic testing
(b) Magnetic resonance imaging of the cervical spine
(c) Magnetic resonance imaging of the shoulder
(d) Computed tomography myelogram

A

(a) Electrodiagnostic testing at this point would be an appropriate test, given that the patient is 3 weeks
postinjury. It could help differentiate a pre- versus postdorsal root ganglion lesion. Although
radiologic studies can assess for possible root avulsion they do not assess the physiologic function
of the peripheral neural elements.

99
Q
Individuals with spinal cord injury who are at the highest risk of developing bladder cancer have as a
risk factor
(a) multiple urinary tract infections.
(b) indwelling Foley catheter.
(c) history of bladder calculi.
(d) ASIA class A.
A

(b) Bladder cancer is the fifth most common neoplasm and the twelfth leading cause of cancer
mortality in the United States. Known risk factors for bladder cancer include male gender, smoking,
occupational exposure to aromatic amines and schistosomiasis. In studies of spinal cord injury
featuring age-matched and gender-adjusted standardized data, however, bladder cancer has
generally been found to be far more prevalent. When looking at independent variables, which point
to a higher risk of bladder cancer, only bladder management method and age at spinal cord injury
significantly predicted bladder cancer. ASIA classification, level of spinal cord injury and a history
of bladder calculi did not contribute significantly. Risk of bladder cancer is the highest in
individuals who have used an indwelling catheters for longer than 10 years. The relative risk of
bladder cancer from indwelling catheter use is relatively unchanged when adjusted for smoking
status. Multiple urinary tract infections is not a risk factor.

100
Q

You are consulted on a 25-year-old patient with cardiomyopathy who has expressed interest in
cardiac rehabilitation. In reviewing her echocardiogram, you note that her left ventricular ejection
fraction (LVEF) is 20%. During your discussion with the house staff you maintain that
(a) exercise capacity is directly correlated with LVEF in patients with heart failure.
(b) an LVEF less than or equal to 20% is a relative contraindication to cardiac rehabilitation.
(c) patients with LVEF less than 30% may attain work capacities of 100 watts.
(d) cardiac rehabilitation may proceed, but should be confined to resistive exercise.

A

(c) In a large series, cardiac patients with LVEFs less than 30% had normal hemodynamics and 28%
attained a work capacity of 100 watts. Thirty-three percent of patients with LVEFs greater than
45% had abnormal hemodynamic responses and lower work capacities. These data indicate that
perfomance capacity is largely dependent on peripheral adaptations and that LVEF has limited
predictive capacity.

101
Q

In a patient with early generalized myasthenia gravis which electromyographic abnormality is the
most likely?
(a) Fibrillation potentials
(b) Fasciculation potentials
(c) Amplitude variability in a single motor unit
(d) Myokymic discharges

A

(c) This can be noted using a trigger and delay line and observing a single motor unit firing
repetitively. The other abnormalities are not seen in myasthenia gravis.

102
Q

Your 10-year old patient with T6 ASIA class B paraplegia complains of right knee pain. On
examination there is no swelling of the knee or leg. Knee examination is normal. The right leg
appears shorter when the hips and knees are flexed. What is the most likely cause of these findings?
(a) Knee sprain
(b) Right hip subluxation
(c) Hip adductor spasticity
(d) Dysesthetic pain

A

(b) Hip subluxation is the most likely cause of knee pain in a child with T6 ASIA B SCI. Pain from
hip pathology is often referred to the knee in children. While hip adductor spasticity contributes to
subluxation or dislocation, the spasticity itself is not painful. Dysesthetic pain is usually
generalized. An abnormal knee examination is usually found in a knee injury that causes pain

103
Q

In an upper-extremity prosthesis, an advantage of a voluntary-closing hook terminal device that a
voluntary-opening device does not have is
(a) the ability to carry objects with less fatigue.
(b) improved ability to manipulate fragile objects.
(c) an adjustable pinch force effected by changing the number of elastic bands.
(d) a better ability to visualize the object being manipulated.

A

(b) One advantage of a voluntary-closing hook terminal device is the ability to adjust the amount of
pinch force by adjusting pressure on the cable control. This improves the ability to manipulate
fragile objects. A voluntary opening terminal device uses elastic bands to determine the pinch force
and the number of bands would have to be reduced to decrease the pinch force. When carrying
objects, voluntary-closing terminal devices require constant cable tension and this can result in
quicker fatigue. The ability to visualize the object being manipulated would not be dependent upon
whether the terminal device is voluntary closing or voluntary opening.

104
Q

A 38-year-old laborer presents with shoulder pain after falling on to the tip of his left shoulder. He
felt immediate pain in the upper part of his shoulder, but no numbness or tingling in his arm. On
examination, he is noted to have a deformity on the superior aspect of his shoulder. He has pain with
horizontal adduction of his left arm across his chest and is having difficulty lifting his left arm. His
passive range of motion is good. Which x-rays with the patient holding and not holding weight(s)
should you order?
(a) A left shoulder anteroposterior radiograph
(b) A scapular Y view radiograph of the left shoulder
(c) Bilateral anteroposterior shoulder radiographs
(d) A left shoulder axillary view radiograph

A

(c) The first step would be to make sure the patient is neurovascularly intact. The next step would be to
obtain bilateral shoulder films with the patient holding and not holding 10-pound weights to
accentuate any laxity present. Bilateral films are necessary as the patient may have lax ligaments in
both shoulders. A scapular Y view is used for shoulder dislocations. An axillary radiograph is used
for shoulder dislocations and will also evaluate the glenoid.

105
Q

Which process results primarily in an axonal, mixed motor, and sensory peripheral neuropathy?

(a) Alcoholic neuropathy
(b) Diabetic neuropathy
(c) Guillain-Barré syndrome
(d) Lead neuropathy

A

(a) Alcohol-induced neuropathy involves both motor and sensory fibers and is typically axonal in
nature rather than demyelinating. Diabetic neuropathy is usually a mixed motor/sensory and mixed
axonal/demyelinating process. Guillain-Barré syndrome is primarily demyelinating. Lead
neuropathy affects primarily motor fibers and is axonal in nature.

106
Q

In spinal cord injury, increased reports of pain are seen more commonly in patients with

(a) complete spinal cord injury.
(b) hypercalcemia.
(c) gunshot wound.
(d) surgical stabilization

A

(c) The prevalence of pain in spinal cord injury (SCI) has been reported to be as high as 94%. It is
reported to interfere with activities of daily living in 5% to 45% of patients with spinal cord injury.
The onset of SCI pain typically occurs within the first year of injury in the majority of patients. Pain
is reported to decrease in intensity and frequency over time. Factors related to self-reported pain
include patients with older age, incomplete spinal cord injury, cauda equina injuries, cervical spinal
cord injury, central cord syndrome, gunshot wound injury, and syringomyelia.

107
Q

You are formulating a comprehensive rehabilitation program for a cachectic patient with chronic
obstructive pulmonary disease. Supplemental feeding in this patient population has been shown to
(a) improve respiratory muscle endurance
(b) minimize bronchodilator requirements
(c) carry little risk of oxygen desaturation.
(d) improve functional residual lung capacity.

A

(a) Supplemental feeding of malnourished patients leads to improved respiratory muscle endurance and
increased respiratory muscle strength. These improvements occur without changes in skeletal
muscle.

108
Q

On the stand in a civil litigation case, a physiatrist asked about an event causing an injury should
(a) decline to answer when the worker has had no objective testing that proves a diagnosis.
(b) answer probable if he/she believes there is more than a 50% chance the event caused the
injury.
(c) answer probable if he/she believes there is less than a 50% chance the event caused the injury.
(d) decline to answer if he/she is not the treating physician.

A

(b) During civil litigation, the physiatrist may be asked to comment on causality of an injury. The
determination that 1 condition caused another condition should be determined on a “reasonable
degree of medical certainty.” In this setting “probable” indicates that the physician believes there is
a greater than 50% chance that condition A caused condition B. The term “possible” indicates that
the physician believes there is a less than 50% chance that 1 event caused another condition.

109
Q

A 27-year-old woman noted the onset of paresthesias in the lateral 3 digits of the right hand 6
months ago. Initially, these symptoms were constant, lasting for about 1 week, and then resolved
spontaneously. Over the 3 weeks prior to this consultation the paresthesias recurred and they are
now present in both the right upper and lower extremities. Electromyogram of the right upper
extremity is normal. Bilateral median, ulnar, and sural sensory nerve conduction studies were
normal. Right peroneal motor nerve conduction study was normal. What electrodiagnostic study
would be most appropriate to perform at this time?
(a) Single-fiber electromyography
(b) Somatosensory evoked potentials
(c) Right median F wave
(d) Electromyogram of the right lower extremity

A

(b) With all of the studies being normal a diagnosis of multiple sclerosis should be considered. Of
those listed the only study that might provide any useful information would be the somatosensory
evoked potentials.

110
Q
What is the primary motion used to open a voluntary-opening terminal device in a body-powered
transradial prosthesis?
(a) Glenohumeral flexion
(b) Glenohumeral extension
(c) Biscapular elevation
(d) Biscapular adduction
A

(a) The control cable for opening the terminal device of a standard transradial prosthesis using a
voluntary-opening terminal device passes posterior to the elbow joint, along the posterior arm, and
attaches to the posterior support strap of a figure-of-8 harness. This arrangement allows
glenohumoral flexion to produce tension in the cable and open the terminal device. Glenohumoral
extension and biscapular adduction decrease tension on the control cable. Biscapular elevation
would not produce cable tension required for terminal device operation.

111
Q

A 39-year-old mail sorter complains of severe right elbow pain for 6 months. He has tried antiinflammatories
without relief. He denies numbness, but does report weakness in his grip. On
examination, the patient has severe pain with palpation just inferior to the lateral epicondyle. Which
of the following findings will most likely be found on further examination?
(a) Pain will be increased with ulnar deviation of the wrist with resisted flexion.
(b) The patient’s brachioradialis reflex will be significantly diminished or absent.
(c) Resisted wrist extension with a straightened elbow will reproduce the patient’s pain.
(d) Atrophy of the extensor digitorum communis and extensor indicis proprius will be present.

A

(c) This patient has lateral epicondylitis or “tennis elbow,” a condition brought on by repetitive flexionextension
or pronation-supination of the forearm. The pain will be increased by resisted wrist
extension with the elbow at 180°. The reflexes will not be affected, nor will atrophy be noted. This
is not a neurologic condition, but a myofascial one.

112
Q
Which genetic factor may link Alzheimer disease and chronic residual deficits in traumatic brain
injury?
(a) Apolipoprotein-Î4
(b) Human leukocyte antigen B27
(c) Mitochondrial protein C450
(d) Platelet aggregation factor
A

(a) The apolipoprotein-Î4 has been linked both to Alzheimer disease and to chronic effects of
traumatic brain injury.

113
Q

Extraskeletal manifestations of ankylosing spondylitis include

(a) Achilles enthesopathy.
(b) tricuspid regurgitation.
(c) cataracts.
(d) nail pitting.

A

(a) Extraskeletal manifestations of ankylosing spondylitis include uveitis/iritis, aortic regurgitation, and
enthesopathy

114
Q

Regarding Bell’s palsy, which statement is TRUE?

(a) It usually develops over days.
(b) It can recur in a small percentage of patients.
(c) Women are more commonly affected than men.
(d) Bilateral cases frequently occur.

A

(b) Most patients with Bell’s palsy usually present with sudden unilateral facial weakness. Persons of
any age can be affected. Men and women are affected equally and in about 2%–9% of patients the
condition can be recurrent. Very rarely it affects both the facial nerves.

115
Q

A 27-year-old man with a T12 ASIA A spinal cord injury for 10 years presents with right shoulder
pain that is worse with use, particularly when reaching and doing transfers. He plays basketball
twice weekly. Recommendations should include
(a) no wheeling or transfers for 2 weeks.
(b) immobilization of the elbow and shoulder.
(c) electrodiagnostic study of the upper extremity.
(d) strengthening of the scapular stabilizers.

A

(d) Shoulder and neck pain are common following spinal cord injury (SCI). The pain may arise from
the neck, shoulder, girdle, or the glenohumeral joint. Pain may be a symptom of post-traumatic
syringomyelia or a manifestation of cervical disc degeneration. The prevalence of shoulder pain in
persons with SCI is estimated to be 30% to 50%. Rotator cuff tear, bursitis, tendonitis and
impingement have all been reported. While the diagnosis of these disorders is similar to that in the
able-bodied population, the treatment is not. In a person with SCI and upper limb pain, rest is often
not possible. Pain is often related to overall posture and poor biomechanics. Strengthening of
scapular stabilizers can help to correct this imbalance. Immobilization should be avoided. Pain
relief is the focus, and may include: relative rest (not to interfere with a person’s independence),
medications, injections, icing, ultrasound, transcutaneous electrical nerve stimulation, and/or
accupuncture.

116
Q
In a patient being evaluated for pulmonary rehabilitation, which sign suggests restrictive lung
disease?
(a) Plentiful sputum production
(b) Pursed lip breathing
(c) Paradoxical breathing
(d) Hyperresonant lung sounds
A

(c) Patients with restrictive lung disease, contingent on the underlying etiology, will rely on purely
diaphragmatic or paradoxical breathing. Sputum production, auxillary muscle use, and
hyperresonant lung sounds are characteristic of patients with predominant oxygenation impairment.
Pursed lip breathing is used to prevent end expiratory airway collapse

117
Q

Maximum medical improvement is defined as the time when the worker

(a) has completed 3 months of physical therapy.
(b) is not able to perform his/her job because of pain.
(c) is not likely to achieve further improvement of the injury.
(d) is able to return to his/her previous job.

A

(c) Maximum medical improvement is defined as the time when no improvement is likely to further be
achieved after a work-related injury. This definition is exclusive of the amount of time spent in
physical therapy, the worker’s subjective complaints, or whether or not the worker is able to return
to his/her previous job. A worker may reach maximum medical improvement and continue to have
impairments that prevent return to his/her former job.

118
Q

Five days after a motorcycle accident a 25-year-old man presents with an insensate right thumb and
index finger and right upper extremity weakness. Electrodiagnostic studies (with normal values in
parentheses) are as follows:
Nerve Conduction Studies
Motor
Nerve Distal Latency (ms) Amplitude (mV) Conduction Velocity (m/s)
R. Median 3.8 (5) 50 (>45)
R. Ulnar 3.5 (5) 55 (>45)
L. Median 3.4 (5) 54 (>45)
Sensory
Nerve Peak Latency (ms) Amplitude (uV)
R. Median (index) 3.6 (10)
R. Ulnar (little) 3.4 (10)
L. Median (index) 3.5 (10)
EMG
Muscle Fibrillations PositiveWaves Motor Unit Size Recruitment
R. Deltoid 0 0 Normal Mod Decreased
R. Biceps 0 0 Normal Mod Decreased
R. Pronator Teres 0 0 Normal Mod Decreased
R. Triceps 0 0 Normal Normal
R. First dorsal interosseous 0 0 Normal Normal
R. Abductor pollicis brevis 0 0 Normal Normal
R. Paraspinals
C2-4 0 0
C5-7 0 0
C8 0 0
What is the most likely diagnosis?
(a) C6 radiculopathy
(b) Upper trunk brachial plexopathy
(c) Lateral cord brachial plexopathy
(d) No conclusions, since the studies were performed too early

A

(a) The presence of an insensate thumb and index finger with a normal median sensory nerve action
potential (SNAP) recorded from the index finger is indicative of a lesion proximal to the dorsal root
ganglion, such as root avulsion. Positive waves in the paraspinals are usually seen 1 week post
onset. While the full EMG findings may not be apparent for 3 to 4 weeks, the data presented are
sufficient to draw conclusions.

119
Q

A 16-year-old girl who had a severe traumatic brain injury 4 weeks ago with left frontal contusion
and left basilar skull fracture demonstrates worse auditory than visual attention. Which diagnostic
test would be most likely to explain this finding?
(a) Magnetic resonance imaging
(b) Electroencephalogram
(c) Audiogram
(d) Visual evoked response

A

(c) A basilar skull fracture is often associated with a permanent sensorineural hearing loss. Other
traumatic brain injuries may be associated with hearing loss, but this is less common. All patients
with basilar skull fracture should be assumed to have a hearing loss and have audiological
evaluation as soon as possible.

120
Q

Regarding acquired upper extremity amputation in adults,

(a) the most common level is transhumeral.
(b) they account for approximately 50% of all acquired major limb amputations.
(c) they are most common in the 20- to 40-year-old age group.
(d) their most common cause is malignancy.

A

(c) Trauma is the most common cause of upper extremity amputation in adults. Malignancy is a more
common cause of amputation in the pediatric population. Upper extremity amputations occur most
commonly in the 20- to 40-year-old age group, as a result of trauma and work related accidents.
The dominant extremity is affected more commonly and amputations at the transradial level are the
most common level of upper extremity amputation. Upper extremity amputations occur much less
frequently than lower extremity amputations. Upper extremity amputations account for
approximately 20% of all major limb amputations

121
Q

The DeLorme axiom states that
(a) low-weight high-repetition exercises build strength, and high-weight low-repetition exercises
build endurance.
(b) high-weight low-repetition exercises build strength, and low-weight high-repetition exercises
build endurance.
(c) low-weight low-repetition exercises build endurance, and high-weight high-repetition
exercises build strength.
(d) high-weight high-repetition exercises build endurance, and low-weight low-repetition
exercises build strength.

A

(b) The DeLorme axiom states that high-weight, low-repetition exercises build strength and lowweight,
high-repetition exercises build endurance.

122
Q

Regarding subarachnoid hemorrhages,

(a) arteriovenous malformations present with hemorrhage after age 40.
(b) the risk of rebleeding after an unoperated aneurysm is low.
(c) their clinical presentation is nonspecific.
(d) aneurysms usually occur in the anterior region of the circle of Willis.

A

(d) Clinical presentation is not nonspecific. Patients often complain of severe headaches (“worst of
their lives”) and present with loss of consciousness. Atriovenous malformation present with
hemorrhages earlier in life, in the second or third decade. Aneurysms are most commonly found in
the anterior region of the circle of Willis, particularly near branches of the anterior communicating,
internal carotid, and middle cerebral arteries.

123
Q

What is the World Health Organization definition of osteoporosis?

(a) Bone mass 2.5 standard deviations below normal
(b) Bone mass 2.0 standard deviations below normal
(c) Bone mass 1.5 standard deviations below normal
(d) Bone mass 1.0 standard deviation below normal with history of fracture

A

(a) The World Health Organization defines osteoporosis as bone mass 2.5 standard deviations below
normals. A history of fracture is not part of the definition.

124
Q

A 37-year-old woman with polymyositis complains of difficulty walking and occasional falling. On
physical examination, she has muscle strength: iliopsoas 2/5, gluteus maximus 4/5, quadriceps 4+/5,
hamstrings 3+/5, anterior tibialis 4/5, gastrocnemius/soleus 4/5. Range of motion is normal. You
would prescribe
(a) bilateral forearm crutches.
(b) knee-ankle-foot orthoses.
(c) reciprocal gait orthosis.
(d) standard adult walker.

A

(a) The patient’s main weakness is proximal and she has sufficient ankle dorsiflexion and knee
extension strength. Hence she would not benefit just with knee-ankle-foot orthoses. Reciprocal
gait orthoses are designed to help children with active hip flexion but no hip extension.
Furthermore, both these options would be too bulky for such a patient to handle efficiently. A
standard walker would result in a bent forward posture and would limit the patient’s gait speed.
Forearm crutches would allow a more upright posture and an alternate 2-point gait or a 4-point gait.

125
Q

Indications for bullet removal after a gunshot wound injury to the spinal cord include

(a) cauda equina location.
(b) the need to reduce the risk of lead toxicity.
(c) pain reduction.
(d) the need to reduce risk of infection.

A

(a) The role of surgery is controversial in fractures of the thoracic spine and thoracolumbar junction.
Patients with complete injuries do not have significantly improved neurologic function after
decompression. However, individuals with incomplete injuries may benefit from surgical
intervention. Indications for emergent surgical treatment include progressive neurologic
deterioration, and expanding epidural hematoma. Because it may destabilize the spine,
laminectomy is infrequently indicated in acute spinal cord injury. Initial management of penetrating
spinal injury differs from that of other traumatic spinal cord injury. Operative treatment is often not
warranted since neither bullet removal nor spinal debridement has been associated with a lower risk
of infection, diminished pain, or enhanced neurologic recovery. The risk of lead toxicity rarely
necessitates bullet removal. Indications for surgery after penetrating injury include persistent
cerebral spinal fluid leak, early progressive neurologic deterioration with bone, missile fragments,
or hematoma in the spinal canal, and injuries involving the cauda equina.

126
Q

A patient with Kugelberg-Welander disease is hospitalized with acute respiratory failure. Work-up
reveals the patient to have peak cough flows less than 270L/min. After successfully weaning the
patient from ventilatory support, in order to minimize future episodes of acute respiratory failure,
you instruct the patient in
(a) manually assisted cough.
(b) air shifting.
(c) pursed lip breathing.
(d) inspiratory resistive loading.

A

(a) During otherwise benign upper respiratory tract infections, patients with predominantly ventilatory
impairment generally develop acute respiratory failure due to ineffective coughing. When peak
cough flows are less than 300L/min patients should be taught maximal insufflation techniques

127
Q

Which financial barrier impedes return to work?

(a) Partial salary and potential monetary gain through litigation
(b) Lack of a retirement plan at work
(c) Low salary prior to the injury
(d) lack of employer-provided health insurance

A

(a) A worker may have 2 financial disincentives to return to work. First, employees usually receive a
portion of their salary while unable to work, if the injury occurred while the worker was performing
his/her job. Second, the worker may not want to return to work if monetary gain is being sought
through pending litigation.

128
Q

Studying F-wave responses would be most helpful in making an early diagnosis of

(a) cervical radiculopathy.
(b) myotonic dystrophy.
(c) multiple sclerosis.
(d) Guillain-Barré syndrome.

A

(d) Guillain-Barré syndrome (acute idiopathic demyelinating polyradiculoneuropathy) commonly
affects the most proximal portion of the nerve while sparing the main nerve trunk in the early
stages. While motor conduction velocities obtained with ordinary techniques may be normal, Fwave
studies help detect proximal abnormalities.

129
Q

What condition casues the typical “myopathic gait” seen in a young boy with Duchenne muscular
dystrophy with accentuated lumbar lordosis and toe walking?
(a) Hip and knee extensor weakness
(b) Hip flexion and ankle plantar flexion contractures
(c) Hip extensor weakness and plantar flexion contracture
(d) Hip flexion contracture and knee extensor weakness

A

(a) The typical “myopathic gait” seen in early Duchenne muscular dystrophy is caused by weakness of
the gluteus maximus and quadriceps muscles. In order to maintain upright posture the child
assumes the hyperlordotic stance. Contractures of the gastrocsoleus and iliopsoas muscles occur
later in the disorder.

130
Q

One advantage of a small diameter caster (front wheelchair wheel) is

(a) greater ability to traverse rough terrain.
(b) better stability on steep inclines.
(c) less frequent maintenance.
(d) greater maneuverability

A

(d) Small diameter casters on a wheelchair reduce the chair’s turning radius, which gives it greater
maneuverability. These casters frequently are mounted on wheelchairs used for sports such as
basketball. Smaller diameter casters are less well suited for outdoor activities over rough terrain.
The size of the caster does not affect the maintenance required and does not improve wheelchair
stability on inclines. Hard, narrow casters are typically recommended for mobility over smooth,
level surfaces whereas wider, larger casters are better for mobility over uneven, rough surfaces.

131
Q

In 1997, the American Academy of Neurology (AAN) published its report on management of
concussion in sports. They made recommendations regarding return to play after a single sportsrelated
concussion, after the patient is asymptomatic at rest and with exertion. Which
recommendation is appropriate according to the AAN report defining concussion grades?
(a) Grade 1 concussion, the participant may return to play after 15 minutes.
(b) Grade 2 concussion, the participant may return to play after 2 weeks.
(c) Grade 3 concussion, the participant may return to play after 3 weeks.
(d) Grade 4 concussion, the participant may not return to play at all.

A

(a) A Grade 1 concussion has no loss of consciousness (LOC) and symptoms which resolve in less than
15 minutes. Participants may return to play only after being asymptomatic with normal neurologic
examination at rest and with exercise after 15 minutes provided there has been only 1 insult. A
Grade 2 concussion has no LOC, but symptoms last more than 15 minutes. These participants may
return to play after 1 week after a single insult (after 2 weeks for multiple insults). A Grade 3
concussion is characterized by LOC of any duration, either for seconds (brief) or minutes
(prolonged). This participant may return to play after 1 week following a brief LOC, 2 weeks after
prolonged LOC, and 1 month or longer after multiple insults with LOC. There is no Grade 4
concussion in the American Academy of Neurology grading system.

132
Q

Regarding post-traumatic seizures following brain injury,

(a) by definition, early seizures occur in the first 24 hours after an injury.
(b) prophylaxis beyond the first week postinjury does not prevent late seizures.
(c) an association exists between late post-traumatic seizures and alcohol use.
(d) most seizures are of the generalized tonicoclonic type.

A

(b) Early seizures occur within the first week. Late seizures are either simple partial or complex partial.
Alcohol is not a risk factor for developing late post-traumatic seizures.

133
Q

Which pharmacologic agent is a secondary cause of osteoporosis?

(a) Heparin
(b) Diltiazem
(c) Naproxen
(d) Hydrochlorothiazide

A

(a) There are many secondary causes of osteoporosis. Drug induced osteoporosis is seen with alcohol,
heparin, steroids, phenytoin and tobacco. Other causes include immobilization, multiple myeloma,
and endocrine related syndromes such as hyperthyroidism, hyperparathyroidism, and diabetes
mellitus type II.

134
Q

Regarding diabetic peripheral neuropathy, which statement is TRUE?

(a) Autonomic fiber involvement is rare.
(b) Motor fibers are affected before sensory fibers.
(c) Prognosis for recovery is good with adequate serum glucose control.
(d) Both axons and myelin are affected.

A

(d) Diabetes-related peripheral neuropathy involves both motor and sensory fibers and both the axon
and myelin are affected. Sensory fibers are the earliest to be affected and autonomic involvement is
common. Tight serum glucose control limits progression of the neuropathy, but does not help
recovery.

135
Q

You are caring for a patient with a T3ASIA class A spinal cord injury who complains of burning
pain in his legs. Additional review of systems includes urinary leakage between catheterizations,
sexual dysfunction, and difficulty sleeping. The best pharmacologic intervention at this time would
be
(a) amitriptyline (Elavil).
(b) paroxetine (Paxil).
(c) trazodone (Desyrel).
(d) fluoxetine (Prozac).

A

(a) Amitriptyline, a tricyclic antidepressant is among the classic first line treatments in neuropathic
pain. Most common side effects related to tricyclic antidepressants are related mainly to the
anticholinergic effects and include dry mouth, urinary retention, and sedation. For this patient who
has difficulty sleeping, as well as urinary leakage between catheterizations, the anticholinergic sideeffects
may prove to be of benefit. Trazodone has not been demonstrated to reduce pain in for
spinal cord injury. Paroxetine causes insomnia and sexual dysfunction and therefore would not be
appropriate in this patient. Venlafaxine, sertraline, and fluoxetine have proven to be of limited
benefit for neuropathic pain.

136
Q

A 45-year-old breast cancer patient complains of diminished exertional tolerance. She completed
adjuvant chemotherapy 1 year ago. Chest x-ray reveals evidence of congestive heart failure. Which
agents could have caused this patient’s symptoms?
(a) Cyclophosphamide (Cytoxan) and methotrexate (Rheumatrix)
(b) Docetaxel (Taxotere) and bleomycin (Blenoxane)
(c) Cisplatinum (Platinal AQ) and 5-fluorouracil (5-FU)
(d) Doxorubicin (Adriamycin) and herceptin (Trastuzumab)

A

(d) Doxorubicin (Adriamycin) and herceptin (Trastizumab) are increasingly common antineoplastics
used in primary breast cancer treatment. Both are associated with cardiac toxicity. Patients who
receive either agent have a multigated angiogram (MUGA) scan prior to the initiation of
chemotherapy to establish baseline cardiac function. Signs or symptoms suggestive of cardiac
failure indicate the need for a repeat MUGA scan and referral to a cardiologist

137
Q

What ergonomic recommendation is appropriate for a computer user who has excessive wrist
extension at the keyboard?
(a) Adjust the chair height and support the forearms horizontal to the floor.
(b) Raise the height of the mouse and position it on a mouse pad.
(c) Adjust the chair height to angle the forearms towards the keyboard.
(d) Lower the chair height and use an angled foot support.

A

(a) The worker at the computer keyboard may need several adjustments to avoid excessive wrist
extension. Raising the chair height to allow the forearm to remain horizontal to the floor allows the
wrist to remain in a neutral position. Increasing the height of the mouse, angling the forearms
towards the keyboard, and lowering the chair will all increase wrist extension.

138
Q

In electromyographic testing of the ulnar nerve for entrapment at the elbow, the primary reason for
flexing the elbow to 45° or more is that this position
(a) causes the ulnar nerve to be more superficial and thus easier to stimulate.
(b) allows more accurate assessment of the length of the nerve.
(c) allows for the least possible stimulus overflow to the radial nerve.
(d) causes increased compression of the nerve, thus improving the diagnostic yield.

A

(b) For testing of ulnar conduction around the elbow, flexion of 45° or greater is preferred because
conduction studies frequently show slowing in the elbow segment when the fully extended position
is used. The discrepancy is thought to arise from the underestimation of the actual length of the
nerve when using surface distance measurement.

139
Q

In which type of cerebral palsy is a seizure disorder most commonly seen?

(a) Tetraplegia
(b) Diplegia
(c) Athetosis
(d) Hemiplegia

A

(d) Approximately 70% of children with hemiplegia have seizures. About 50% of children with
tetraplegic cerebral palsy have seizures. Seizures are rare in children with diplegia or athetosis.

140
Q

Which shoe modification would NOT be appropriate for an individual with loss of protective
sensation on the foot secondary to diabetes?
(a) Wide toe box
(b) Rocker bottom
(c) Soft inner shoe liner
(d) Solid steel shank

A

(d) All of the shoe modifications listed would be appropriate for an individual with loss of protective
sensation except for the use of a solid steel shank built into the sole of the shoe. This modification
would decrease the accommodative properties of the shoe and may increase the potential for skin
breakdown.

141
Q

Muscle energy technique is a manual medicine treatment that is used to

(a) stabilize unstable segments in the spine.
(b) strengthen painful muscles by isotonic contraction.
(c) increase functional range of motion.
(d) improve efficiency of muscular oxygen extraction.

A

(c) Muscle energy technique is used to increase mobility of a hypomobile segment, increase functional
range of motion, allow the return of symmetrical motion to affected segments, strengthen weakened
muscles, and lengthen contracted or spastic muscles. It uses isometric contractions, not isotonic
ones. It is not used for unstable segments, and does not improve proprioception or focus on
facilitating volitional activity.

142
Q

Which characteristic is a risk factor for heterotopic ossification in traumatic brain injury?

(a) Male gender
(b) Flaccid tone
(c) Long bone fractures
(d) Older age

A

(c) Risk factors for HO include: long bone fractures, spasticity, prolonged immobilization, and
prolonged coma

143
Q
The presence of which factor would give a patient a poor prognosis for home discharge after hip
fracture?
(a) Being 85 years old
(b) Having coronary artery disease
(c) Being of male gender
(d) Having a nonhealing wound
A

(a) Factors associated with permanent institutionalization after hip fractures are age greater than 80
years, lack of involvement by family members, insufficient physical therapy at a skilled nursing
facility, and pre-existing dementia.

144
Q

A 50-year-old woman presents with a 3-month history of progressive swallowing problems. She has
developed left leg weakness. She denies any sensory problems or bowel or bladder incontinence. On
examination, she has extensor plantar responses. What test would be an appropriate next step?
(a) Lumbar puncture
(b) Trial of pyridostigmine
(c) Brain stem auditory responses
(d) Electrodiagnostic studies

A

(d) The patient has symptoms suggestive of upper motor neuron involvement, given extensor plantar
responses. This finding would point away from a neuromuscular junction disorder such as
myasthenia gravis. These symptoms could indicate multiple sclerosis but the clinical picture
described is more typical of amyotrophic lateral sclerosis. Needle exam of the muscles can help
confirm lower motor neuron findings.

145
Q

You are performing urodynamic studies on an individual with a T10 ASIA class A spinal cord injury
sustained 2 years previously. He performs intermittent catheterization every 6 hours and reports no
episodes of urinary incontinence between catheterizations. You find that the filling pressure or leak
point pressure is 20cm of water. At this time, you recommend
(a) oxybutinin (Ditropan).
(b) initiation of Credé maneuver.
(c) continuation of current bladder program.
(d) urecholine (Bethanechol).

A

(c) The primary risk factor for serious urologic complications such as vesicoureteral reflux and
deterioration of renal function in persons with detrusor external sphincter dyssynergia is that of
elevated intravesical pressure. For this reason, it is essential to determine urinary storage and leak
point pressures. In general, damage to the upper urinary tract can be avoided if voiding pressure is
maintained less than 60cm of water while the maximum filling pressure or leak point pressure
should be less than 40cm of water. For this patient who is having no episodes of leaking between
his every 6-hour catheterizations, continuation of current program is reasonable. If on follow-up,
his leak point pressure exceeds 40cm of water, conversion to a low pressure system via
anticholinergics would be reasonable. The use of a cholinergic agonist (Urecholine) would be
contraindicated, since this agent may increase the intravesical pressure.

146
Q

A 35-year-old AIDS patient is referred to your clinic for evaluation of mild cognitive deficits and
lower extremity weakness. She was diagnosed with AIDS 5 years ago and her most recent CD4
count was 190/ml. On mental status examination you note decreases in attention, concentration, and
short-term memory, with relatively preserved insight. Her motor examination is significant for
cogwheel rigidity, ataxia, and tremor. What is most likely etiology of these patients clinical findings?
(a) Toxoplasmosis
(b) Cryptococcal meningitis
(c) AIDS dementia complex
(d) Primary central nervous system lymphoma

A

(c) The patient’s presentation includes virtually all of the clinical characteristics of AIDS dementia
complex (ADC). This is a relentlessly progressive encephalopathy whose presentation is typical of
subcortical dementia. It is a late complication of AIDS, usually occurring when the antigenic
marker on hilpir T cells (the CD4 count) falls below 250/ml. While each of the other choices can
complicate AIDS, it would be unusual for them to present with the constellation of signs and
symptoms seen in this patient.

147
Q

What is the most common cause of ulnar neuropathy in computer users?

(a) Repetitive forearm supination from manipulation the mouse
(b) Compression of the ulnar nerve at the elbow by the arm rest
(c) Excessive elbow extension due to a mouse placed out of reach
(d) Repetitive forearm pronation with elbow extension from mispositioned keyboard

A

(b) Compression of the ulnar nerve at the elbow is a common occurrence among computer users.
Etiologies include compression of the nerve against the chair armrest or combined elbow flexion
with pronation. Elbow extension and supination reduce tension on the ulnar nerve at the elbow.

148
Q

Which of the following is an axonal peripheral neuropathy?

(a) Hereditary motor and sensory neuropathy type I
(b) Diphtheria neuropathy
(c) Metachromatic leukodystrophy
(d) Vincristine neuropathy

A

(d) Vincristine can cause an axonal peripheral neuropathy. The others listed are demyelinating
peripheral neuropathies.

149
Q

A 1-year-old child with a midlength transfemoral limb deficiency presents for prosthetic

management. Which component should be in the prosthetic prescription?
(a) Socket with a growth liner
(b) Single action knee joint
(c) Vertical shock pylon
(d) Dynamic foot

A

(a) The 1-year-old child should be fit with a simple prosthesis which suspends securely and allows for
growth. The knee joint should be added between 3 and 5 years. The SACH foot is most commonly
prescribed because of its simple design and durability. All prostheses for growing children should
incorporate a removable growth liner in the socket.

150
Q

In which case would a wrist-hand-finger orthosis utilizing dynamic metacarpophalangeal extension
assistance be most indicated?
(a) Complete radial nerve injury at the level of the mid-humerus
(b) Partial ulnar nerve injury at the level of the elbow
(c) Complete median nerve injury in the forearm
(d) C5 level ASIA class A spinal cord injury

A

(a) With a radial nerve injury at the level described, there is loss of wrist and metacarpal extension
because of denervation of the forearm extensor muscles. A dorsal wrist-hand-finger orthosis that
holds the wrist, fingers, and thumb in extension and permits flexion of the digits with a low-profile
outrigger is used to substitute for this loss of function. This type of orthosis would be inappropriate
for the other conditions described.